You are on page 1of 25

Desgloses comentados

Pediatra
T1
P141

P184
Neonatologa

MIR 2008-2009

MIR 2011-2012

Hoy en da, es poco frecuente diagnosticar clnicamente un hipotiroidismo congnito debido a que, en su mayor parte, son diagnosticados con el cribado neonatal, que se realiza determinando los niveles de TSH obtenida entre los dos y los cinco das de vida (cuyos valores estaran elevados). Las manifestaciones del hipotiroidismo congnito tardan varias semanas en presentarse plenamente, estando totalmente establecidas hacia el 3-6 mes de vida. Aparece una facies peculiar (cara tosca, prpados y labios tumefactos, nariz corta con base deprimida, hipertelorismo, boca semiabierta y macroglosia). Otras manifestaciones tpicas son el retraso en la maduracin sea, la hernia umbilical, la ictericia prolongada, la letargia y el estreimiento. El principal problema del hipotiroidismo congnito es su repercusin sobre el desarrollo intelectual. ste sera ms grave cuanto ms se demore el diagnstico (respuesta 3 correcta), por lo que es importante instaurar el tratamiento cuanto antes. Aunque la causa ms frecuente de hipotiroidismo congnito es la disgenesia tiroidea, independientemente de cul sea la causa exacta, ante cualquier caso de hipotiroidismo debemos apresurarnos a establecer el tratamiento sustitutivo lo antes posible.

La enfermedad de las membranas hialinas o sndrome de distrs respiratorio del recin nacido prematuro es debida a la inmadurez pulmonar por dficit de surfactante pulmonar. ste, fabricado por los neumocitos tipo II a partir de las 20-24 semanas de edad gestacional, es un agente tensioactivo que permite la apertura de los alvelos y, por tanto, un adecuado intercambio gaseoso. Est formado por protenas, colesterol, fosfatidilcolina, esfingomielina y, fundamentalmente, por lecitina, cuya cantidad va aumentando segn avanza el embarazo. La madurez pulmonar se alcanza finalmente a las 34-35 semanas de edad gestacional, de manera que por debajo de esta edad gestacional, la falta de surfactante provoca un aumento de la tensin superficial que determina el colapso alveolar y un shunt intrapulmonar (alvelos perfundidos, pero no ventilados). Clnicamente, observaremos un pretrmino con signos de dificultad respiratoria en las primeras horas de vida, con cianosis, mala respuesta al oxgeno, hipercapnia, acidosis respiratoria y crepitantes a la auscultacin. En la radiografa de trax observamos un parnquima retculo nodular fino, tambin denominado patrn en vidrio esmerilado, con broncograma areo y escaso volumen pulmonar. El tratamiento de eleccin es la administracin de surfactante exgeno por va endotraqueal, garantizando adems un soporte respiratorio adecuado, evitando la hiperoxia, que es mal tolerada por el pretrmino (uno de los principales factores de riesgo para desarrollar retinopata de prematuro). La principal complicacin a largo plazo es la evolucin a una displasia broncopulmonar. La administracin de corticoides prenatales a la madre, ante la amenaza de parto prematuro, disminuye su incidencia y su gravedad.

P187

MIR 2008-2009

P179

MIR 2008-2009

La cocana no suele producir sndrome de abstinencia en el recin nacido, a diferencia de los opiceos. Sin embargo, pueden aparecer complicaciones obsttricas, como el parto pretrmino, el desprendimiento prematuro de placenta, etc. Por otra parte, puede afectar directamente al feto, manifestndose como CIR, microcefalia, hemorragias intracraneales, anomalas digestivas o renales, muerte sbita, alteraciones conductuales o neurolgicas. La respuesta correcta, por tanto, es la 3. Lo que produce la cocana no es parto postrmino, sino todo lo contrario: pretrmino.

Un caso clnico complicado, donde resulta bastante difcil decidirse entre las opciones 2 y 5. Las otras son relativamente fciles de descartar, por las razones siguientes: R1: no existen signos de hemorragia intraventricular, que normalmente cursara con bradicardia, hipoventilacin, convulsiones, hipotona y fontanelas a tensin (slo coincide la hipotona, que es un dato inespecfico). R3: la fenilcetonuria es una enfermedad metablica en la que se produce una acumulacin anormal de fenilalanina. Se manifiesta varias semanas despus del nacimiento. La mayora de estos enfermos muestran piel clara, ojos azules y color claro del pelo (no puede sintetizarse la melanina, ya que la fenilalanina es un precursor suyo y no puede metabolizarse en el hgado). Es tpico el retraso mental por acmulo de fenilalanina en plasma hasta 30 veces por encima del nivel normal. R4: la enfermedad de la membrana hialina producira una imagen de condensacin pulmonar, con broncograma areo y habra alteraciones en la auscultacin pulmonar (en este caso, nos dicen que ventila sin problemas). La verdadera dificultad estara en distinguir la hipoglucemia de la sepsis neonatal. Al ser hijo de madre diabtica tiene un claro factor de riesgo para padecer una hipoglucemia. Sin embargo, al ser un recin nacido pretrmino, tambin lo tiene para sepsis. Por tanto, debemos guiarnos por las manifestaciones clnicas.

Desgloses comentados

497

La presencia de acrocianosis y malestar general, as como el color terroso, podran aparecer en ambos casos. Sin embargo, cuando se trata de una hipoglucemia, lo habitual sera la aparicin de temblores o algn otro tipo de clnica neurolgica (irritabilidad, convulsiones) que en este caso faltan. Como en esta ocasin predominan las manifestaciones inespecficas, sin foco infeccioso claramente identificable, resulta ms probable la respuesta 2 que la 5.

Pediatra
MIR 2007-2008 P181 MIR 2005-2006

P185

El estudio placentario (opcin 1 correcta), a nivel histolgico, puede mostrar datos como amnionitis, infartos, hipervascularizacin, abruptio E incluso puede cultivarse. Muchas veces, la clnica no es especfica y se manifiesta como CIR, citopenias, ictericia precoz, e incluso de forma asintomtica (opciones 3 y 4 correctas). Este estado asintomtico puede acarrear secuelas irreversibles. En concreto, la forma de presentacin ms frecuente de la toxoplasmosis es una coriorretinitis silente que puede abocar a la ceguera en la edad adulta. Por ello, todos los recin nacidos infectados han de ser tratados con sulfadiacina, pirimetamina y cido folnico durante el primer ao de vida (opcin 5 falsa). Asimismo, fjate que las respuestas 1, 3 y 4 tienen un puede. En Medicina, ya sabes que nada es seguro. Es muy difcil que unas opciones tan poco categricas resulten falsas.

Pregunta asequible acerca del estudio de infecciones connatales en el recin nacido, si bien haca aos que no aparecan en el MIR. Las tres primeras opciones tocan aspectos generales del manejo de estas infecciones. En todas, es til el estudio serolgico del RN (opcin 2 correcta), especialmente de la IgM especfica al germen, ya que es de sntesis fetal (recuerda que es pentamrica y que por sus dimensiones no cruza placenta). Puedes repasar todo lo comentado en la siguiente tabla. P185 (MIR 07-08) Infecciones connatales

Pregunta de dificultad moderada acerca del retraso en la eliminacin del meconio. En condiciones normales, el RN expulsa el meconio en las primeras 24-36 horas de vida. Si la eliminacin de meconio no se produce, se acumula una masa compacta a escala anorrectal que impide la expulsin. Son causas de retraso en la eliminacin de meconio: 1. El sndrome de colon izquierdo hipoplsico (frecuente en hijos de madre diabtica). 2. Fibrosis qustica. 3. Aganglionosis rectal. 4. Drogadiccin materna. 5. Prematuridad. 6. Tratamiento con sulfato de magnesio de la preeclampsia materna. El hipotiroidismo congnito no cursa en principio con retraso en la eliminacin del meconio, porque la clnica de instauracin es tarda, estando completamente establecida a los tres o seis meses de edad. Son datos sugestivos: una facies peculiar (cara tosca con prpados y labios tumefactos, nariz corta con base deprimida, hipertelorismo con boca abierta y macroglosia), estreimiento, hernia umbilical, ictericia prolongada, letargia y un retraso en la maduracin sea. Sin embargo, hoy en da el diagnstico clnico es poco corriente gracias a las pruebas de screening precoz mediante la determinacin de la TSH en muestra de sangre obtenida a las 48 h y a los cinco das de vida de todo RN. Unos niveles de TSH menores de 10 mcUI/ml son compatibles con la normalidad, mientras que si la TSH supera los 50 mcUI/ml el caso es compatible con un hipotiroidismo primario congnito y debe instaurarse precozmente tratamiento con levotiroxina con el objetivo de mejorar el pronstico neurolgico de estos pacientes. La causa ms frecuente de hipotiroidismo congnito es la disgenesia tiroidea, que incluye tiroides aplsicos, hipoplsicos y ectpicos, siendo esta ltima la causa ms comn de hipotiroidismo congnito. En la primera plantilla de respuestas que dio el Ministerio se consideraba como respuesta correcta la opcin 5; no obstante, posteriormente la pregunta fue anulada, ya que algunos autores consideran que el hipotiroidismo congnito puede estar tambin implicado en el retraso en la eliminacin del meconio.

P168

MIR 2004-2005

Pregunta difcil sobre los efectos de los corticoides. La que no debe plantearte ningn tipo de duda es la 5, puesto que se indican con este fin, acelerar la sntesis pulmonar de surfactante. Adems, reducen la incidencia de

498

Desgloses comentados

hemorragia de la matriz germinal (respuesta 3 correcta), de NEC (respuesta 4 correcta), de DAP y de neumotrax.El resto de las opciones no deben preocuparte. La opcin 1 es falsa, porque los corticoides nicamente aumentan el riesgo de infeccin cuando se emplean crnicamente, produciendo en tal caso inmunodepresin celular. Pero el uso puntual de corticoides, aunque sea una dosis relativamente alta, carece de este efecto.

Pediatra
P192 (MIR 04-05) Enterocolitis necrotizante. Radiologa

P170

MIR 2004-2005

La incompatibilidad Rh requiere que una madre, Rh negativa, sea puesta en contacto previamente con sangre Rh positiva. En cambio, la incompatibilidad ABO puede afectar al primer hijo sin necesidad de sensibilizacin previa (respuesta 1 correcta). Esta circunstancia es ms frecuente (respuesta 5 falsa), pero menos grave, que la incompatibilidad Rh, por lo que no se asocia a anemias fetales severas (respuestas 2 y 3 falsas). El test de Coombs indirecto es positivo y el directo tambin suele serlo, salvo en raras excepciones, de modo que la opcin 4 tambin se podra considerar correcta. En consecuencia, estamos ante una pregunta que podra impugnarse.

P170 (MIR 04-05) Isoinmunizacin del recin nacido (RN)

P168

MIR 2003-2004

P192

MIR 2004-2005

La enterocolitis necrotizante suelen presentarla en el MIR como un cuadro de distensin abdominal y deposiciones sanguinolentas. Sin embargo, esta pregunta va un poco ms all. Observa que nos dan tres antecedentes que implicaran un mayor riesgo para esta enfermedad: prematuridad, bajo peso, necesidad de ventilacin asistida... Si a esto aadimos un cuadro clnico compatible con obstruccin intestinal (estreimiento, vmitos, distensin y dolor clico), la opcin que mejor encaja es la 1. De hecho, una de las complicaciones ms habituales de esta entidad son las estenosis intestinales, que generan cuadros suboclusivos, como en este lactante que nos comentan.

Pregunta de dificultad moderada. Teniendo en cuenta la clnica descrita en el caso sera razonable dudar entre las dos ltimas opciones, pero considerando los antecedentes expuestos, no queda opcin a duda. La vitamina E es una vitamina liposoluble cuya deficiencia aparece unida a enfermedades malabsortivas graves y prolongadas: fibrosis qustica, colestasis (y dentro de stas la atresia de vas biliares), etc. La carencia nutricional no se ha descrito. Se manifiesta por un sndrome potencialmente reversible caracterizado por ataxia (cerebelosa y por afectacin de cordones posteriores) y neuropata perifrica. Con respecto al resto de opciones, es interesante comentar la 5. Esta patologa debuta en la infancia o adolescencia con ataxia progresiva de la marcha y, ya que su diagnstico es eminentemente clnico (apoyndonos en los antecedentes familiares, EMG, etc.), es fundamental descartar previamente un dficit de vitamina E. Las opciones 1 y 3 son incorrectas, puesto que en el caso no se describe ninguna alteracin del estado mental. El dficit de vitamina A cursa con ceguera nocturna, xeroftalmia, retraso del crecimiento, apata, sequedad cutnea, etc. Como comentario final resear la importancia de la reposicin peridica de vitaminas liposolubles en las colestasis neonatales.

Desgloses comentados

499

P191

Pediatra
MIR 2002-2003 MIR 2002-2003 P178 MIR 2001-2002

Pregunta fcil sobre el distrs respiratorio del recin nacido. Mediante este caso clnico se nos exponen las principales caractersticas de lo que se conoce como: sndrome de Avery, pulmn humedo, taquipnea transitoria del recin nacido o sndrome de distrs respiratorio tipo II. Epidemiologa: recin nacidos a trmino o postrmino por cesrea o por parto vaginal muy rpido. Patogenia: retraso en la absorcin del lquido de los pulmones. Clnica: distrs respiratorio que se valora mediante el test de Silverman (regla mnemotcnica: DIRE QUE ATLETIC: DI-sociacin traxabdomen; RE-traccin xifoidea; QUE-jido respiratorio; ALE-teo nasal y TI-raje). Mejora con la administracin de oxgeno. Diagnstico: Rx de trax: lquido en las cisuras, hiperinsuflacin, diafragmas aplanados, derrame pleural, no broncograma; GAB: normal. Tratamiento: atmsfera enriquecida en oxgeno, recuperacin en uno o dos das. Por eso, la opcin correcta es la 1. La opcin 2 es el tratamiento de la enfermedad de membrana hialina o sndrome de distrs respiratorio tipo I (con la cual se podra plantear duda, porque es tpico de recin nacidos pretrmino, pero su imagen radiolgica tpica es la afectacin bibasal con infiltrado retculo-nodular y broncograma).

P192

Pregunta de dificultad media sobre un tema que es muy preguntado, como es la enterocolitis necrotizante. La enterocolitis necrotizante es una lesin isqumico-necrtica que suele afectar a leon distal y colon proximal y que con frecuencia se acompaa de sepsis bacteriana (opcin 1 incorrecta). Etiologa: es tpica de recin nacidos de muy bajo peso con enfermedades graves, policitemia, alimentacin muy precoz y con elevados volmenes, hipoxia... No obstante, parece que la lactancia materna puede ser un factor protector (tambin lo es de la retinopata de la prematuridad) (opcin 3 correcta). Se ha involucrado a diversos patgenos (E. coli, C. perfringens, rotavirus y S. epidermidis), pero en la mayora de los casos no se detecta ningn germen responsable. (opcin 2 incorrecta). Clnica: repentina distensin abdominal, retencin gstrica y deposiciones sanguinolentas en la 2 semana. Diagnstico: laboratorio: leucopenia, neutropenia, trombopenia (opcin 4 incorrecta), hiponatremia. Rx de abdomen: signos ms precoces son el edema de asas y el asa fija; patrn en miga de pan, neumatosis intestinal (signo diagnstico), neumoperitoneo (si hay perforacin) y gas en vena porta. Tratamiento: dieta absoluta, fluidoterapia y descompresin nasogstrica; antibiticos; tratamiento quirrgico si hay perforacin, peritonitis o sepsis refractaria al tratamiento mdico.

mas y, a veces, derrame pleural. No existe broncograma areo (respuesta 4 correcta). La enfermedad de la membrana hialina (EMH) se produce por un dficit de surfactante pulmonar, lo que provoca un aumento de la tensin superficial y colapso de los alveolos. Afecta, sobre todo, a RNPT (dado que los niveles de surfactante no alcanzan la madurez pulmonar hasta las 35 semanas de edad gestacional). Cursa con un distrs respiratorio precoz moderado-severo, con cianosis progresiva que con frecuencia no mejora con la oxigenoterapia. En la gasometra hay hipoxemia, hipercapnia y acidosis metablica y/o respiratoria, y en la radiografa es tpico (pese a que no patognomnico, pues una radiografa similar se puede ver en la sepsis neonatal) un infiltrado reticulogranular con broncograma areo. El sndrome de aspiracin de meconio (SAM) es un cuadro tpico de RN postrmino que ha padecido un sufrimiento fetal agudo. La gravedad clnica depende de lo espeso del meconio. El RN presenta distrs respiratorio y signos de atrapamiento areo, con hiperinsuflacin torcica y riesgo de extravasacin extrapulmonar de aire, dando como complicaciones neumotrax, neumomediastino o neumopericardio. El neumomediastino suele ser asintomtico, apareciendo en algunos casos una distensin de las venas del cuello, por estar dificultado el retorno venoso. Los tonos cardacos estn atenuados. La radiografa tpica del SAM es un infiltrado parcheado, de aspecto algodonoso, con diafragmas aplanados. A veces tambin neumotrax o neumomediastino. El sndrome de Wilson-Mikity es un cuadro clnico radiolgicamente similar a la displasia broncopulmonar (cuadro producido al someter a neonatos a ciertos procedimientos teraputicos, como la oxigenoterapia a altas concentraciones) que se describi en RNPT sin antecedentes de EMH.

La meningitis en el recin nacido puede estar causada por bacterias, virus, hongos o protozoos. La meningitis puede presentarse dentro de una sepsis o como una infeccin focal. Las causas bacterianas ms normales de meningitis neonatal son los estreptococos del grupo B, E. coli y Listeria. Tambin pueden producir meningitis otros estreptococos, cepas no tipificables de H. influenzae y estafilococos, entre otros. P178 (MIR 01-02) Sepsis y meningitis neonatal

P177

MIR 2001-2002

Existen mltiples causas de dificultad respiratoria en el neonato. La taquipnea transitoria del recin nacido se produce por un retraso en la absorcin del lquido de los pulmones fetales. Aparece en RNT o RNPT nacidos por cesrea o parto vaginal rpido. Clnicamente, el neonato presenta distrs respiratorio leve-moderado, de inicio precoz y que mejora con pequeas cantidades de oxgeno. La gasometra no suele mostrar alteraciones significativas y en la radiografa se observa un aumento de las marcas vasculares, lquido en las cisuras, aplanamiento de los diafrag-

500

Desgloses comentados

P177 (MIR 01-02) Distrs respiratorio

Pediatra

La duda razonable est entre las opciones 4 y 5, dado que ambos pueden causar meningitis neonatal. Sin embargo, el estafilococo es uno de los grmenes implicados en la sepsis de origen nosocomial, sobre todo en neonatos que han estado cateterizados. El cuadro clnico consiste inicialmente en una sepsis sin foco, apareciendo con frecuencia, ms tarde, focalidad menngea (respuesta 4 incorrecta; respuesta 5 correcta). Las manifestaciones clnicas de la meningitis neonatal pueden ser inicialmente inespecficas, con o sin signos neurolgicos (letargia, fontanela abombada, convulsiones...). El diagnstico se confirma mediante el examen del LCR y la identificacin del microorganismo. El LCR se considera patolgico si hay pleocitosis (ms de 30 clulas/mm3, con polimorfonucleares mayor del 60 %), hiperproteinorraquia (mayor de 150 mg/100 ml) e hipoglucorraquia (glucosa menor de 30 mg/dl).El tratamiento de la meningitis del RN no ingresado se establece empricamente con ampicilina ms cefotaxima, y en el RN ingresado con vancomicina ms ceftacidima +/ anfotericina B.

rostoma de Kasai. Las enfermedades metablicas del hgado, entre ellas el dficit de alfa 1- antitripsina, que es la ms usual, y la colestasis familiar le siguen en frecuencia. En los centros de trasplante se utilizan varios sistemas de puntuacin para predecir la mortalidad por hepatopata, pero los factores ms destacados son la prolongacin del tiempo de tromboplastina parcial, la ascitis, el aumento de los niveles de bilirrubina indirecta y la disminucin del colesterol. Algunos trastornos, como la atresia biliar sin respuesta a la ciruga de Kasai, deben corregirse con trasplante antes de que aparezcan complicaciones tardas. El tratamiento previo al trasplante resulta decisivo para el xito de la intervencin. Los aspectos ms relevantes comprenden la nutricin, las vitaminas, la vacunacin y la atencin sanitaria general del nio. El tratamiento mdico tambin comprende el control de la hipertensin portal. El estado del paciente en el momento del trasplante, a excepcin de los casos de coma profundo, no influye en la supervivencia, pero s afecta al perodo de recuperacin y a las complicaciones. El pronstico para los supervivientes es muy favorable. El crecimiento mejora y los estigmas de hepatopata crnica desaparecen. Repasa las indicaciones de trasplante con la siguietne tabla.

P185

MIR 2001-2002

El trasplante heptico representa el tratamiento convencional de las hepatopatas terminales de la infancia. La indicacin ms frecuente en los nios es la atresia biliar extraheptica que no responde a la portoente-

Desgloses comentados

501

P185 (MIR 01-02) Indicaciones del trasplante de hgado en la infancia Indicaciones N de casos

Pediatra
145 42 27 7 4 4 19 16 13 12 7 5 3 2 2 4 270

Atresia biliar Enfermedad metablica del hgado - Dficit de -1 antitripsina - Tirosinemia - Enfermedad de Wilson - Otras Colestasis familiar Insuficiencia heptica fulminante Hipoplasia biliar Hepatitis Cirrosis idioptica Hepatitis neonatal Colangitis esclerosante Fibrosis heptica congnita Carcinoma Miscelanea Total

Los datos que hay que tener en cuenta para evaluar una talla baja son la velocidad de crecimiento y la correlacin con la talla de los padres. En este caso clnico se trata de un nio con una velocidad de crecimiento normal. Por otra parte, la talla se puede considerar baja pero dentro de los lmites normales y, sobre todo, acorde con la talla materna. Adems, nos explican que en la rama paterna, existen antecedentes de pubertad retrasada. Por consiguiente, no est indicado utilizar hormona del crecimiento ni realizar estudios del eje hipotlamo-hipofisario. Tampoco cabra esperar lo que dice la respuesta 3 (de hecho, sera al contrario: edad sea retrasada con respecto a la edad cronolgica). En otras palabras, no estamos ante un caso en el que haya que pensar en patologa de ningn tipo, al menos con estos datos, por lo que no precisa que tomemos medidas extraordinarias

P188

MIR 2008-2009

Una pregunta aparentemente difcil sobre el desarrollo psicomotor que resulta mucho ms sencilla si la analizas con lgica. La duda razonable estara entre las opciones 2 y 3. De hecho, en esta convocatoria, a la mayor parte de los opositores les sonaba la idea de los seis meses. La dificultad est en decidirse por una u otra. En el Manual CTO, la solucin apareca claramente, y citamos de forma textual: 6 mes: inicia la sedestacin, que se completa a los 8 meses. Por tanto, si la sedestacin se inicia en el 6 mes, difcilmente habr un 50% de los lactantes que ya puedan sentarse antes de ese momento. La respuesta correcta sera entonces la 3. Por otra parte, ante una duda como sta, es posible que, en el momento del examen, no dispongamos de ningn argumento terico y haya que arriesgar entre dos opciones. En tal caso, ante una pregunta cuyas cinco opciones son numricas, la apuesta ms segura sera el valor intermedio, ordenndolos todos de menor a mayor. En realidad, as es como muchos opositores ganaron el punto de esta pregunta.

De Ziell BJ, Gartner JC, Malatack JJ, y cols.: liver transplantation in children: A pediatricians perspective. Pediatr Ann 20:691,1191.

T2
P152

Desarrollo y nutricin

MIR 2010-2011

Pregunta sobre el desarrollo psicomotor del nio. En Pediatra son tpicas estas preguntas, son bastante difciles y especficas, ya que los periodos que nos proponen son muy concretos, y existen diversos hitos del crecimiento y desarrollo que pueden solaparse en diferentes momentos del tiempo. La prueba ms empleada es el test de Denver, que valora los aspectos personal, social, lenguaje y motricidad fina y grosera en los nios desde el nacimiento hasta los 6 aos. La respuesta correcta es la opcin 3. Corre, sube escaleras andando cogido de la mano, construye una torre de cuatro cubos, se sienta en sillas pequeas, dice 10 palabras (de media), identifica una o ms partes del cuerpo, come solo, pide ayuda en caso de necesitarla, etc.

P181

MIR 2007-2008

El crecimiento y desarrollo en pediatra es un tema complejo debido a su amplitud. Para el MIR, solamente es necesario conocer un pequeo resumen. Esta pregunta se anul. Durante la primera semana de vida el recin nacido (RN) puede sufrir una prdida de peso fisiolgica en torno al 5-10% con respecto al nacimiento, resultado de la excrecin del exceso del lquido extravascular y la ingesta escasa. Por eso, la opcin 2 es falsa, puesto que no suele disminuir ms del 10% (prdida de peso patolgica). A medida que la madre mejora en la tcnica de la lactancia y el calostro es reemplazado por una leche ms rica en grasas, el RN recupera peso, alrededor de 30 g/da durante el primer mes (opcin 3 correcta). As, al final del primer ao de vida, el peso al nacimiento aproximadamente se triplica y la talla se incrementa un 50% con respecto al nacimiento (si al nacer mide 50 cm, al final del primer ao medira 75 cm). Por ello, la opcin 1 sera tambin falsa, dado que la duplicacin de la talla ocurre alrededor de los cuatro aos de vida. Las opciones 4 y 5 son correctas y hacen referencia al desarrollo psicomotor. Como norma, la mayora de los reflejos arcaicos desaparecen en torno a los cuatro meses de vida. No olvides que el desarrollo psicomotor sigue un patrn cefalocaudal, y uno de los primeros hitos que aparecen es la sonrisa social hacia el 1,5-2 meses de vida.

P166

MIR 2009-2010

Una pregunta de dificultad media que se responde de forma directa a partir del Manual CTO. Tal como explicamos en el Captulo 2 (Desarrollo y nutricin) de la Seccin de Pediatra, el indicador ms sensible para detectar precozmente este tipo de alteraciones es la velocidad de crecimiento (respuesta 2 correcta). sta es mxima durante los cuatro primeros aos de la vida, producindose un segundo incremento (aunque menor) durante la pubertad, siendo ms precoz en nias que en nios.

502

Desgloses comentados

P190

Pediatra
MIR 2007-2008
P188 (MIR 05-06) Tipos de deshidratacin
Prdida Agua = solutos Solutos > agua <270 mosm/l <130 mEq/l Osmolaridad 285 mosm/l Na 130-150 mEq/l

Una pregunta relativamente sencilla sobre la talla baja, ya que el sndrome de Klinefelter produce precisamente lo contrario: talla alta, sobre todo a expensas de la mitad inferior del cuerpo. Para acertar esta pregunta, en realidad no era necesario conocer rarezas como las que enuncian las respuestas 3 y 4. El sndrome de Seckel, entre otras manifestaciones, ocasiona microcefalia, bajo peso al nacer y talla baja. El de Silver-Russell, que es causa de talla baja, tambin se asocia con malformaciones esquelticas (asimetra) y alteraciones morfolgicas del crneo. Los sndromes de Down y Turner son ms conocidos como causas de talla baja. Recuerda que, por el contrario, el sndrome de Klinefelter (XXY), igual que el sndrome XYY, produce talla alta.

de una acidosis metablica: por un lado el HCO3 est bajo (11) y, por otro, la clnica de diarrea y vmitos. Fjate que la pCO2 lejos de estar aumentada como correspondera a una acidosis respiratoria, est descendida (pCO2 de 25); esto se explica por la compensacin que est realizando el pulmn eliminando CO2 del organismo para corregir de esta forma el pH (recuerda estos binomios: hipoventilacin-acidosis/hiperventilacin-alcalosis).

ISOTNICA HIPOTNICA HIPERTNICA Agua > solutos >300 mosm/l >150 mEq/l

P180

MIR 2006-2007

Con respecto al dficit de hormona de crecimiento en la infancia, los pacientes que aquejan dicho dficit muestran una velocidad de crecimiento inferior a la normal y su curva de crecimiento se desva progresivamente del canal normal. Pero si el dficit es congnito, se manifiesta a partir de los 6-12 meses de edad, siendo la talla y el peso al nacer normales. Eso es debido a que en esta etapa de la vida el crecimiento no depende de GH, sino de la insulina. La mayora responden al tratamiento con GH sinttica con una aceleracin de la velocidad de crecimiento hasta un lmite normal o, incluso, por encima de lo normal.

Mucosas secas Pliegue + Fontanela deprimida Clnica Hipotensin Oliguria

Mucosas secas + Pliegue ++ Mucosas secas ++ Fontanela deprimida + Sed Hipotensin + NRL (hemorragia subdural) Oliguria + Convulsiones

P171

MIR 2003-2004

P188

MIR 2005-2006

Aunque sea aparentemente compleja, esta pregunta se puede contestar fcilmente conociendo la definicin de talla baja o hipocrecimiento. Se habla de hipocrecimiento cuando la talla se encuentra por debajo del percentil 3 para la edad y sexo. La paciente del caso tiene una talla en el P10, con lo que en principio debemos asumir que su crecimiento es normal (opcin 1 correcta). Los dems datos del enunciado no hacen sino apoyar este diagnstico. La paciente es una nia de diez aos que todava no ha iniciado el desarrollo puberal (estadio 1 de Tanner). Se considera que en el perodo prepuberal inmediato la velocidad de crecimiento debe ser al menos de 4 cm/ao (la de la paciente es de 5 y, por consiguiente, normal). Por otro lado, la talla gentica de la paciente (determinada por la talla media de los progenitores) est en el P15, bastante prxima a su talla real. Finalmente, se asume que la edad sea de un paciente es normal cuando se encuentra en un intervalo de +/ 1 ao en torno a la edad cronolgica.

Pregunta fcil de un tema constante en el MIR: repsala porque no puedes permitirte fallarla. Se trata, simplemente, de valorar una analtica general en un paciente peditrico. nicamente hemos de determinar el tipo de deshidratacin (isotnica, hipotnica o hipertnica) y el tipo de acidosis (respiratoria, metablica o mixta). Para contestar a este tipo de preguntas basta con conocer los niveles normales de sodio, pCO2 y bicarbonato. En general, qudate con las cifras 35-45, puesto que te sern tiles para varios de estos niveles: Na: 135-145 mEq/l. pCO2: 35 - 45 mmHg (aprovecho para recordarte los niveles del potasio que son 3,5-5, pero para recordarlos utiliza 3,5-4,5). El pH normal: 7,35-7,45. El bicarbonato oscila alrededor de 25 (21-28), pero en el MIR te darn niveles suficientemente extremos como para que decidas si est alto, bajo o normal. Sabiendo esto veamos los datos que nos ofrecen para responder la pregunta: El Na es de 137, por tanto, est comprendido entre los valores normales: se trata entonces de una deshidratacin isotnica. El pH es de 7,20, consecuentemente, el paciente se encuentra en acidosis. El tipo de acidosis viene determinada por los niveles de pCO2 y de HCO3. Si el paciente no ventila adecuadamente (un ejemplo clsico del MIR sera la intoxicacin por sustancias sedantes), la pCO2 aumentar y hablaremos de acidosis respiratoria. Si el paciente est perdiendo HCO3 por alguna va (el ejemplo tpico sera el de la pregunta, con diarrea y vmitos; pero tambin existen causas renales como la acidosis tubular proximal o tipo II), ste disminuir dando lugar a una acidosis conocida como metablica. En el caso de nuestra pregunta tenemos varios datos para decir que se trata

P176

MIR 2003-2004

Pregunta relativamente fcil si se tienen en cuenta las nociones bsicas de alimentacin del lactante. Hemos de fijarnos en el matiz que dan a lactancia materna en la respuesta 2: prolongada. El RN tiene unos depsitos de hierro suficientes para los primeros meses de vida. Esto hace que la lactancia materna sea suficiente para evitar la aparicin de anemia ferropnica en ese perodo. Pero cuando stos empiezan a agotarse, a pesar de la buena absorcin del hierro de la leche materna, ste debe aportarse de forma extra mediante alimentos enriquecidos (como frmulas de continuacin). Tambin es conveniente en esta pregunta recordar que tras los primeros das de vida, la causa ms comn de anemia es el dficit de hierro.

Desgloses comentados

503

P197

Pediatra
MIR 2002-2003 MIR 2001-2002 P190 MIR 2008-2009

Pregunta fcil sobre la alimentacin complementaria del lactante. Sobre la alimentacin complementaria lo ms importante es que se debe iniciar a partir del 4-6 mes; no antes porque el lactante todava tiene problemas de deglucin, supone una sobrecarga de solutos y el aparato digestivo todava no est preparado; pero tampoco despus, porque la leche sola ya no cubre las necesidades del lactante. Lo primero que se debe introducir son los cereales sin gluten y se debe esperar al 6-8 mes para comenzar con los cereales con gluten (por eso las opciones 3 y 5 son falsas). En cuanto al resto de opciones, en la 1 ya hemos comentado que precisamente esos cereales son los que no tienen gluten; la opcin 2 no se ha relacionado con la dermatitis atpica, su introduccin precoz s que se ha relacionado con formas graves de celaca; la opcin 4 nos habla de la hidrlisis mediada por la amilasa pancretica que no influye para nada sobre los cereales. Por eso fue anulada, ya que todas las opciones son falsas.

El paciente de la pregunta cumple con varios criterios: tos seca espasmdica de dos semanas de evolucin, que incluso le han provocado hemorragias subconjuntivales por el esfuerzo. El agente microbiolgico principal es la Bordetella pertussis, si bien tambin se han aislado Bordetella parapertussis y Bordetella bronchiseptica, aunque en menor proporcin. La tosferina es una de las enfermedades transmisibles ms contagiosas, siendo los lactantes menores de 6 meses, los adolescentes (como el caso de la pregunta) y los adultos los ms susceptibles de padecer la enfermedad. Estos ltimos actan como transmisores al lactante, donde provoca mayor morbilidad. Normalmente, la tos ferina del adulto cursa como un cuadro catarral inespecfico. Pese a que el paciente est correctamente vacunado, se sabe que la inmunogenicidad de la vacuna es relativamente variable. La eficacia frente a la enfermedad clnica es del 70-90% durante 2-5 aos, y va disminuyendo con el tiempo, no prolongndose ms all de 12 aos tras la administracin de la ltima dosis de vacuna. El paciente recibi la ltima dosis de tos ferina a los 6 aos, con lo cual la inmunidad esperada es menor. Por todo ello, la respuesta correcta es la opcin 4: un macrlido, como tratamiento ms eficaz frente al gnero Bordetella. La opcin 1 (asma) se descarta porque la auscultacin es normal y no hay signos de dificultad respiratoria. Seran precisos ms estudios complementarios (opcin 2 falsa) como muestras para aislamiento de Bordetella en una muestra clnica. Habra que estudiar a los contactos (opcin 3 falsa) para descartar contagios. Por ltimo, la hemorragia conjuntival (opcin 5) es un dato tpico. Se debe a la hiperpresin endotorcica, secundaria a los accesos de tos, que se transmite hacia los vasos conjuntivales.

P190

La muerte es un proceso que, obviamente, el nio vive de manera diferente al adulto. Ante la separacin definitiva (la muerte de un progenitor, por ejemplo) la mayora de los nios preadolescentes no parecen sufrir el tpico proceso de duelo. El duelo del nio puede estar enmascarado por una conducta que no es tpica de los adultos. Muchos nios continan sus actividades cotidianas y el principal mecanismo de defensa ante la catstrofe es la negacin, tanto manifiesta como inconsciente, mantenida por el deseo y la esperanza mgicos de reaparicin y de reencuentro. Algunos nios muestran sentimientos hostiles y de ira hacia el progenitor que sobrevive, y tienden a identificarse con el progenitor perdido y a idealizarlo, a veces con fantasas de reencuentro que acompaan a la negacin. Otra posibilidad es que los nios demuestren un dolor considerable al morir uno de los padres, o tras un lapso de tiempo, una vez que el mecanismo de defensa de la negacin deja de ser eficaz. Los nios menores de cinco aos creen que la muerte es reversible, y posiblemente piensan que los muertos vuelven a la vida o en los fantasmas. En la fase siguiente, hasta los ocho o nueve aos, la muerte se personifica, como por ejemplo "el Coco" que castiga y se venga. Slo pasada esta edad el nio comprende la muerte de forma realista, como un proceso biolgico final y universal (respuesta 3 correcta).

Una pregunta laboriosa por la extensin del caso clnico pero, en realidad, sencilla y referente a un tema caracterstico del Examen MIR. Aunque la definicin de bronquiolitis habla de lactantes menores de dos aos con su primer episodio de dificultad respiratoria, aparece ms en los menores de seis meses, como sucede en esta pregunta. Es importante tener en cuenta los sntomas previos para su diagnstico, porque en los lactantes de ms edad, sobre todo con antecedentes familiares u otros factores de riesgo para hiperreactividad bronquial (dermatitis atpica, alergias alimentarias, tabaquismo pasivo), la bronquiolitis puede confundirse con el asma. Las crisis de broncospasmo, a diferencia de las bronquiolitis, suelen tener una aparicin brusca; en la auscultacin se escuchan fundamentalmente sibilancias (sin ruidos de secreciones), y lo ms importante, las crisis se repiten con frecuencia. Adems, en este caso nos mencionan un episodio catarral previo, que tambin respalda la respuesta 2.

T3
P173

Aparato respiratorio

P192 MIR 2009-2010

MIR 2008-2009

La OMS define como caso clnico de tosferina a aquel sujeto con tos que dura al menos dos semanas, con uno o ms de los siguientes sntomas: Accesos de tos paroxstica. Estertor al inspirar (gallo). Vmitos postusivos y sin otra causa obvia.

El cuadro que nos describen corresponde a una tos ferina. Esta enfermedad es una infeccin producida principalmente por Bordetella pertussis. Afecta preferentemente a menores de un ao. Clnicamente produce un prdromo inespecfico, de tipo catarral, y luego aparece la fase que caracteriza a la enfermedad, que es la de tos paroxstica. Origina accesos repentinos de tos, con series repetitivas de mltiples toses enrgicas, acompandose despus de un ruido inspiratorio que recibe el nombre de gallo, al pasar aire a travs de una glotis cerrada. La tos, en ocasiones, se asocia con vmi-

504

Desgloses comentados

tos (tos emetizante). Las complicaciones de la tos ferina son: Neumona: la ms frecuente. Suele ser por sobreinfeccin bacteriana secundaria. Convulsiones, cuya causa no est bien establecida. Otras: prolapso rectal, hemorragia subconjuntival, hernia umbilical, epistaxis. Estas complicaciones se deben a la intensa presin positiva intratorcica que se produce en relacin con la tos. A pesar de tratarse de una infeccin bacteriana, es caracterstica la presencia de linfocitosis en el hemograma, como podemos ver en este caso. El tratamiento antibitico de la tos ferina es la eritromicina durante dos semanas.

Pediatra
P186 MIR 2005-2006
Pregunta de dificultad fcil-media en forma de caso clnico de bronquiolitis. La bronquiolitis es una enfermedad viral que cursa con obstruccin inflamatoria de las pequeas vas areas. Se define como el primer episodio de dificultad respiratoria con sibilantes espiratorios que tiene lugar en un nio < 2 aos (el nio de la pregunta tiene dos meses), con sntomas de infeccin de va respiratoria de tipo vrica. El virus respiratorio sincitial (VRS) es el agente causal ms frecuente. La fuente de infeccin suele ser familiar y la transmisin se produce por va respiratoria. Los nios mayores y adultos no presentan dificultad respiratoria a pesar de la infeccin, dado que toleran mejor el edema bronquiolar. En efecto, la contribucin de la va area de pequeo calibre a la resistencia total es mayor en los lactantes, y durante una infeccin por el VRS se produce una obstruccin bronquiolar causada por edema, acmulo de moco y detritus celulares, con el resultado final de disminucin del radio de la va area y aumento reflejo de la resistencia al paso de aire. Es frecuente el antecedente de infeccin respiratoria leve unos das antes (hace tres das tuvo mocos, tos, estornudos, etc.) para aparecer posteriormente tos, dificultad respiratoria e irritabilidad. Habitualmente no existe fiebre, pero puede haber febrcula. Debido a la taquipnea, el lactante puede manifestar rechazo del alimento. En la exploracin encontramos signos de dificultad respiratoria como aleteo nasal, tiraje, etc. En la auscultacin se oye una espiracin alargada y sibilancias. La disminucin significativa de los ruidos respiratorios indica obstruccin casi completa de las vas areas y constituye, por tanto, un signo de gravedad. El diagnstico es habitualmente clnico apoyado en la radiologa. Se asume que bronquiolitis es todo aquel primer episodio de dificultad respiratoria baja que sufre un lactante menor de dos aos, por otro lado, sano. La radiologa muestra hiperinsuflacin pulmonar. Se puede demostrar el virus en secreciones nasofarngeas por inmunofluorescencia y elevacin de los ttulos de anticuerpos en sangre o cultivo. Para el tratamiento es esencial que recuerdes que los broncodilatadores son de muy dudosa eficacia, puesto que la fisiopatologa de la enfermedad no es la broncoconstriccin (como ocurre en el asma). Lo que ms les beneficia es la administracin de oxgeno. La adrenalina inhalada puede ser til al producir vasoconstriccin y, por consiguiente, ayudar a disminuir el edema. La ribavirina inhalada se reserva para casos graves. En resumen, recuerda que no estn indicados ni broncodilatadores, ni corticoides, ni antibiticos ni sedantes. P186 (MIR 05-06) Diagnstico diferencial entre bronquiolitis y asma

P227

MIR 2007-2008

Una pregunta ms sencilla de lo que parece, dado que se presta a la deduccin lgica. Nos preguntan por el virus que no causa neumonas (o lo hace con escasa probabilidad). Entre las opciones que nos exponen, sera lgico pensar que el virus respiratorio sincitial pueda producir infecciones broncopulmonares (de hecho, como sabes, es causa de bronquiolitis). Tambin sabemos que el virus Influenzae (gripe) ocasiona cuadros respiratorios, as que no es de extraar que el parainfluenzae produzca cuadros parecidos. Siguiendo este razonamiento, la duda estara entre el adenovirus y el rotavirus. Sobre ellos, deberas saber: Adenovirus: estos virus producen infecciones respiratorias en nios y reclutas (colectivos cerrados, como campamentos militares, colegios.) y otros muchos posibles cuadros (diarrea, conjuntivitis, faringitis). Rotavirus: es la causa ms comn de diarrea de origen vrico en el lactante, pero es rarsimo que produzca infecciones respiratorias (respuesta 4).

P192

MIR 2006-2007

Con respecto a la fibrosis qustica, el diagnstico se basa en las manifestaciones clnicas tpicas (respiratorias, digestivas y genitourinarias) o antecedentes de fibrosis qustica en un hermano o prueba de cribado neonatal positivo, ms datos de laboratorio de disfuncin de CFTR (test del sudor positivo en dos resultados positivos en das distintos) o dos mutaciones de FQ en estudio gentico o alteraciones en la diferencia de potencial nasal. P192 (MIR 06-07) Diagnstico de fibrosis qustica

Desgloses comentados

505

El principal diagnstico diferencial de la bronquiolitis se debe realizar con el asma. El asma: Es raro en menores de un ao. No hay infeccin previa. Suele existir historia familiar. Las crisis son repetidas (solamente se llama bronquiolitis al primer episodio de dificultad respiratoria de un lactante menor de dos aos). El comienzo es brusco.

Pediatra
P195 MIR 2002-2003 MIR 2002-2003
Pregunta fcil sobre la fibrosis qustica.

En consecuencia, en este caso la opcin correcta sera el tratamiento conservador, puesto que es el primer paso a realizar. Las opciones 1 y 4 son maniobras invasivas que slo se utilizaran en casos ms graves; la opcin 2 hace referencia al tratamiento quirrgico del reflujo gastroesofgico.

P190

Pregunta de dificultad media sobre la patologa congnita del aparato respiratorio. No es un tema muy preguntado en el MIR, pero donde las caractersticas de los distintos tipos de cuadros son muy parecidas. La laringomalacia y la traqueomalacia son las causas ms habituales de estridor congnito. El diagnstico diferencial entre ambas se realiza por el tipo de estridor; as, un estridor inspiratorio es ms tpico de una afectacin de vas altas (larngeo), mientras que si es tanto inspiratorio como espiratorio es tpico de afectacin de vas intermedias (trquea). La afectacin de vas bajas (bronquios, bronquiolos) lo que suele producir son sibilancias espiratorias. P190 (MIR 02-03) Patologa larngea en el nio

Se trata de una pregunta directa sobre un diagnstico ya dado, la fibrosis qustica, y donde nos la describen clnicamente como enfermedad multisistmica caracterizada por obstruccin crnica de las vas respiratorias, siendo la principal causa de enfermedad pulmonar crnica grave en la infancia y tambin la responsable de la mayor parte de las insuficiencias pancreticas exocrinas en las primeras etapas de la vida. Desde el punto de vista gentico, es la enfermedad hereditaria ms frecuente y letal de la raza blanca. Se hereda de forma autosmica recesiva, estando el gen localizado en el brazo largo del cromosoma 7 (una regla para recordarlo es: FQ, si damos la vuelta a la F nos queda 7q). Por eso, la opcin incorrecta es la 3. El resto de opciones nos describen las caractersticas patognicas de la enfermedad. Desde el punto de vista clnico, lo ms preguntado suele ser la frecuencia de sobreinfecciones bacterianas; teniendo en cuenta que el germen ms frecuente: P. aeruginosa, el ms precoz: S. aureus y el ms grave: B. cepacea. Asimismo, la insuficiencia respiratoria crnica lleva al desarrollo en estos pacientes de un cor pulmonale. Desde el punto de vista diagnstico se requieren dos criterios: Manifestaciones clnicas tpicas o antecedentes de FQ en un hermano o pruebas de cribado neonatal positivas, ms algn dato de laboratorio de disfuncin de CFTR. Test de sudor positivo; dos mutaciones conocidas de FQ en estudio gentico; alteracin en la diferencia de potencial nasal. Repasa la figura de la pregunta 192, MIR 06-07. El tratamiento es fundamentalmente sintomtico.

T4
P135

Aparato digestivo

MIR 2011-2012

El diagnstico se establece por laringoscopia directa. En general, el proceso se resuelve espontneamente y no precisa tratamiento especfico. Si hay dificultades para la alimentacin, puede ser necesaria la alimentacin por sonda. Muy excepcionalmente se utiliza la traqueotoma.

Pregunta de dificultad media debido a ser un tema que haca aos no apareca en el MIR, si bien con los contenidos del manual, abordable. Fjate que te presentan un neonato con imposibilidad al paso de sonda nasogstrica en el paritorio. La sospecha de atresia esofgica se confirma si sigues leyendo el enunciado donde menciona bolsn esofgico atrsico. Para filiar ante qu tipo de atresia nos encontramos (figura), nos comentan que la neumatizacin intestinal es NORMAL. Con estos datos excluimos las fstulas tipo I: donde slo existe atresia y al no haber fstula no puede haber paso del aire al intestino; tipo II: la fstula proximal permite el paso nicamente a va respiratoria sin neumatizacin intestinal; y tipo V: donde no

506

Desgloses comentados

existe bolsn esofgico atrsico. Nos quedara la tipo III y la IV, la primera con una frecuencia del 87% y la tipo IV con frecuencia inferior al 1%. De todas maneras esta pregunta no requiere un conocimiento tan profundo, ya que la tipo III es la ms frecuente con diferencia (87%), la que has de saber, ya que es la que con probabilidad te preguntaran (recuerda la regla: la aTRESia esofgica ms frecuente es la tipo TRES). P135 (MIR 11-12) Atresia de esfago

Pediatra
ESTREIMIENTO FUNCIONAL Inicio al nacimiento Enterocolitis Tamao de las heces Retraso ponderal Ampolla rectal Incontinencia fecal Tono del esfnter Raro No Grandes Raro Llena Frecuente Variable Vaca Rara

un retraso en la evacuacin del meconio, que suele precisar estimulaciones para realizar deposicin, que probablemente presentara un retraso ponderal y que en el tacto rectal se apreciara la ampolla vaca y una hipertona del esfnter anal. Sin embargo, nos encontramos ante un lactante que realiza deposiciones de consistencia blanda de forma espontnea sin requerir estimulacin ni laxantes, por lo que probablemente se trate de un estreimiento funcional. El hecho de que las realice cada 5-6 das puede considerarse un ritmo intestinal normal. Por tanto, las opciones que deberamos valorar son la 4 y la 5, aunque nos faltaran datos para contestar una de las dos con certeza. A continuacin, se exponen las principales diferencias entre el estreimiento funcional y la enfermedad de Hirschsprung. P136 (MIR 11-12) Diferencias entre estreimiento funcional y enfermedad de Hirschsprung
ENFERMEDAD DE HIRSCHSPRUNG Frecuente Posible Acintadas o normales Frecuente

Elevado

P153
La prioridad de la ciruga es evitar la neumopata causada por las microaspiraciones de cido gstrico por la fstula distal, cada vez que el neonato inspira ejerciendo presin negativa (opcin 3 correcta). De hecho, hasta que el neonato est estable y se opere, se coloca en decbito prono con objeto de evitarlas. Fjate que las opciones 1 (imposibilidad de deglutir saliva) y 4 (imposibilidad de alimentacin enteral) vienen a ser lo mismo y, por ello, son descartables, adems de que no suponen una prioridad quirrgica. La opcin 2 (malformaciones cardacas frecuentemente asociadas) se excluye con el enunciado tras evaluacin diagnstica que descarta otras anomalas.... En la clnica, hay que tener presente la asociacin VACTERL (malformaciones Vertebrales, Anorrectales, Cardacas, Traqueales y Esofgicas, Renales y radiales -Limb en ingls-) presente en un 30-50% de las atresias esofgicas; de ah que lo mencionen en el enunciado. Por ltimo, recuerda que una de las complicaciones postoperatorias es el reflujo gastroesofgico (MIR 05-06, 191), por anomalas intrnsecas de la funcin esofgica. Otras como traqueomalacia, estenosis esofgica, fstula de la anastomosis o de la fstula traqueoesofgica son ms infrecuentes.

MIR 2010-2011

Pregunta fcil sobre un tema tpico, repetido en varias convocatorias. Este tipo de preguntas no se pueden fallar. El cuadro clnico corresponde a una estenosis hipertrfica del ploro: lactante varn que debuta en torno a los 20 das de vida con vmitos alimentarios (no biliosos), a chorro o proyectivos, que le provocan una alcalosis metablica hipoclormica (se pierde HCl), con tendencia a la hipopotasemia, un estado permanentemente hambriento e irritable, deshidratacin y desnutricin. En la exploracin fsica, se palpa en algunos casos la oliva pilrica. La tcnica de eleccin para confirmar el diagnstico es la ecografa abdominal (respuesta 3 correcta). Cuidado con la respuesta 1, la Rx simple de abdomen es til en el diagnstico diferencial, pudiendo ver una nica burbuja tanto en la hipertrofia como en la atresia pilrica y una doble burbuja en la atresia duodenal, pero nos piden la prueba complementaria de eleccin. El tratamiento preoperatorio consiste en fluidoterapia i.v. para corregir las alteraciones hidroelectrolticas, siendo el tratamiento curativo la piloromiotoma de Ramsted. Hay que tener presente en la patologa digestiva peditrica los diagnsticos diferenciales, en este caso con la atresia pilrica (vmitos no biliosos desde las primeras tomas), la atresia duodenal (vmitos biliosos precoces, regla de la D: duodenal, doble burbuja, sndrome de Down), el reflujo gastroesofgico (vmito atnico de contenido alimentario desde la primera semana de vida, la pHmetra, respuesta 2, es la prueba ms

P136

MIR 2011-2012

En el caso que nos comentan, nos falta informacin tanto de la anamnesis, como de la exploracin fsica, por lo que la pregunta fue anulada por el Ministerio. Ante un nio con estreimiento desde el nacimiento, debemos diferenciar dos causas principalmente: un estreimiento funcional y una enfermedad de Hirschsprung. Si estuviramos ante una enfermedad de Hirschsprung describiran a un paciente que present

Desgloses comentados

507

sensible y especfica pero no la primera a realizar) y con la insuficiencia suprarrenal congnita (vmitos desde el nacimiento con alcalosis metablica hiperpotasmica). P153 (MIR 10-11) Diagnstico diferencial de la patologa digestiva peditrica
PATOLOGA Estenosis hipertrfica de ploro Atresia de ploro Atresia de duodeno Tipo de vmito NO bilioso NO bilioso BILIOSO Radiografa Distensin gstrica con escaso gas distal Imagen de NICA burbuja Imagen de DOBLE burbuja

Pediatra
P180 MIR 2008-2009
Caso clnico tpico sobre una estenosis hipertrfica del ploro.

donde generalmente asienta el tramo aganglinico. Entre las complicaciones probables nos encontramos la de la pregunta: obstruccin. Inicialmente se toman medidas de desobstruccin, hasta la realizacin de pruebas ms especficas que confirmen la sospecha diagnstica (enema, manometra y biopsia): opcin 3.

P170

MIR 2009-2010

Un tema que, desde hace muchas convocatorias, no apareca en el examen MIR (en concreto desde 1996). No obstante, la pregunta es bastante sencilla, ya que la clnica del paciente tiene poco que ver con la que producira una fisura anal (respuesta 1 falsa). Definimos intolerancia alimentaria como aquella reaccin adversa que aparece tras la ingesta de alimentos, que no est mediada por IgE. Los sntomas son exclusivamente digestivos: diarrea, rectorragia, vmitos, dolor abdominal, etc. La alergia alimentaria, a diferencia de la intolerancia, s est mediada por IgE. Clnicamente, aparte de los sntomas digestivos, podremos verlos de tipo alrgico: urticaria, angioedema, broncospasmo e incluso anafilaxia, en los casos ms graves. La leche de vaca es el alimento ms frecuentemente implicado en los procesos de intolerancia y alergia alimentaria. La protena ms relacionada con estos fenmenos es la beta-lactoglobulina, seguida de la alfa-lactoalbmina y la casena. El diagnstico es fundamentalmente clnico. Los sntomas aparecen cuando el nio toma leche de vaca y desaparecen cuando este alimento es excluido de la dieta. El pronstico de esta entidad es favorable, ya que desaparece a lo largo de los primeros aos de la vida. Observa que, en el nio que nos plantean, solamente existen sntomas digestivos, por lo que se tratara de una intolerancia. Por lo tanto, no estara mediada por IgE, como dice la respuesta 3. De hecho, la intolerancia es ms frecuente que la alergia. El tratamiento consiste en sustituir la frmula normal por un hidrolizado de protenas lcteas. Los sntomas alrgicos pueden precisar un manejo especfico, como antihistamnicos e incluso adrenalina subcutnea, si hubiese angioedema. En casos seleccionados se ha planteado el uso de cromoglicato sdico.

El sntoma cardinal en esta enfermedad lo constituyen los vmitos. stos sern no biliosos, proyectivos, y aparecen despus (o casi despus) de las tomas. Suelen comenzar a partir de los 20 das de vida. Debido a la prdida de hidrogeniones y cloruros que supone el vmito, tiende a producirse una alcalosis metablica hipoclormica. En este caso clnico, las pequeas estras sanguinolentas pueden ser debidas a la intensidad del vmito, que puede producir hiperpresin a nivel de los vasos de la mucosa esofgica, dandose algunos y produciendo un leve sangrado. En la exploracin podemos encontrar grados variables de deshidratacin o desnutricin, as como ictericia o subictericia. En algunos casos, incluso es palpable la oliva pilrica a nivel epigstrico, bajo el reborde heptico. La tcnica de eleccin para confirmar el diagnstico es la ecografa abdominal (respuesta 4 correcta). Se detectar un grosor del msculo pilrico por encima de lo normal (> 4 mm), as como un aumento de la longitud global del canal pilrico.

P182

MIR 2008-2009

Una pregunta difcil sobre el diagnstico de la enfermedad celaca. Ante la sospecha de esta enfermedad, lo habitual es realizar una determinacin de anticuerpos (recuerda que los ms sensibles y especficos son los IgA-antitransglutaminasa). Si esta determinacin resulta positiva, estara indicado realizar una biopsia intestinal, que tambin podra plantearse ante una determinacin negativa, pero con una firme sospecha clnica. Por tanto, el papel de los anticuerpos en el diagnstico de esta enfermedad tiene valor fundamentalmente como cribado. La biopsia intestinal es una prueba necesaria para establecer el diagnstico. Se realizara por lo menos en una ocasin, estando el paciente consumiendo gluten. En este caso clnico ya tenemos una biopsia intestinal compatible con enfermedad celaca (la duda estara en la necesidad de realizarla por segunda vez). La confirmacin de la recuperacin histolgica despus de la retirada del gluten (segunda biopsia) se realizara en casos dudosos. No obstante, en este caso lo tenemos bastante claro: buena evolucin clnica despus de retirar el gluten de la dieta y negativizacin de los anticuerpos, por lo que en principio bastara continuar con la dieta.

P186

MIR 2007-2008

P175

MIR 2009-2010

La enfermedad de Hirschprung o aganglionosis colnica es todo un clsico en las preguntas del MIR de Pediatra. Antecedentes tpicos son estreimiento desde el perodo neonatal (tapn meconial) y a lo largo del primer ao de vida (esta paciente tiene 4 meses), debido a la peristalsis inadecuada del segmento aganglinico. Otros signos tpicos seran la distensin abdominal (megacolon) debido a la retencin fecal y ausencia de gas distal,

Pregunta con un caso clnico tpico de reflujo gastroesofgico madurativo o fisiolgico. La manifestacin clnica ms usual es la regurgitacin en la mayora de las tomas a las pocas semanas de vida (la mayora empiezan ya en la primera semana). Dado que el paciente presenta un adecuado desarrollo pondoestatural, hablamos de un reflujo gastroesofgico fisiolgico debido a un esfnter esofgico inferior incompetente por inmadurez (opcin 4 correcta).

508

Desgloses comentados

La estenosis pilrica es un cuadro mucho ms agudo, con vmitos proyectivos no biliosos, que comienza en torno a las tres semanas de vida con riesgo de deshidratacin para el neonato (opcin 2 falsa). Tanto la opcin 1 como la 3 hacen referencia a reacciones adversas frente a la protena de la leche de vaca, y por ello no guardaran tan buena evolucin: se afectaran el peso y la talla. En la intolerancia, es comn el hallazgo de heces con sangre. La alergia puede provocar urticaria, angioedema, broncoespasmo, e incluso anafilaxia como forma grave.

Pediatra
P191 MIR 2005-2006
Se trata de una pregunta de elevada dificultad sobre un tema poco preguntado en el MIR. P191 (MIR 05-06) Atresia esofgica

Por ltimo, la opcin 5, el vlvulo intestinal, se presentara como un cuadro obstructivo y clnica de peritonismo. No permitira una evolucin tan solapada en el tiempo (hablan de dos meses y medio), sino mucho ms aguda.

P188

MIR 2007-2008

La intolerancia a disacridos provoca que stos no se absorban adecuadamente. Mediante un efecto osmtico, atraen agua hacia la luz intestinal, de manera que las heces se vuelven acuosas (diarrea postprandial, opcin 1 correcta). A su vez, al ser fermentados por la flora saprofita del colon, las heces se vuelven cidas (por lo que aparece secundariamente eritema perianal) y se acompaan de meteorismo, debido al gas generado en la fermentacin; as pues, stas se eliminan explosivamente. En esto se fundamenta la prueba diagnstica del H2 espirado (opciones 2, 3 y 4 correctas). Las heces esteatorreicas (grisceas, brillantes, adherentes, flotantes) son propias de los cuadros clnicos en los que existe malabsorcin de lpidos (opcin 5 falsa). En Pediatra, no olvides nunca la celiaqua ni la fibrosis qustica.

P183

MIR 2005-2006

Pregunta de dificultad fcil-media acerca del diagnstico de laboratorio de la intolerancia-malabsorcin de lactosa. La malabsorcin de lactosa se debe a un dficit de lactasa, que puede ser congnito, pero que lo ms comn es que sea secundario a procesos que lesionan de forma difusa el epitelio intestinal (infecciones, enfermedad celaca...). Los disacridos no hidrolizados se acumulan en el intestino, donde las bacterias producen cidos orgnicos e hidrgeno, lo que atrae agua a la luz intestinal y se genera diarrea osmtica. Clnicamente se manifiesta por heces espumosas, con pH bajo, ricas en azcar y que suelen excoriar las nalgas. Se produce tambin distensin abdominal, borborigmos y ocasionalmente dolor clico. Para el diagnstico podemos usar: Deteccin de cuerpos reductores en heces mediante la prueba del clinitest que se cuantifica segn cambio de color. Recuerda que la sacarosa no es un azcar reductor y, por tanto, si se sospecha intolerancia a la sacarosa, previamente debe hidrolizarse con cido clorhdrico. Determinacin de pH fecal inferior a 5,6 o determinacin de cido lctico en heces. Actividad de disacaridasas medida en la biopsia intestinal disminuida. Test de H2 en aire espirado tras sobrecarga oral de lactosa. El test de la ureasa en la biopsia intestinal se usa en el diagnstico de infeccin por H. pylori. Se trata de un mtodo invasivo, puesto que se hace a partir de una biopsia de antro y cuerpo gstrico. Se basa en que la ureasa que origina la bacteria hidroliza la urea y cambia el color de un indicador, lo que significa que hay infeccin por H. pylori. Por consiguiente, nada tiene que ver con el dficit de lactasa.

La atresia y fstula traqueoesofgica ocurren con una incidencia 1/3.0004.500 RN vivos. Existen diversas clasificaciones, aunque la ms aceptada es la clasificacin de Ladd, que distingue cinco tipos: Tipo 1: atresia sin fstula. Tipo 2: fstula proximal y atresia distal. Tipo 3: atresia proximal y fstula distal. Tipo 4: doble fstula. Tipo 5: fstula sin atresia. La forma ms frecuente es la III, con casi el 85% de los casos. Debemos sospechar esta patologa ante: Existencia de polihidramnios. Imposibilidad para pasar sonda nasogstrica en el paritorio. Salivacin excesiva. Cianosis y atragantamiento con las tomas. Si existe una fstula traqueoesofgica distal, aparecer una distensin abdominal importante, mientras que si no existe fstula distal, el abdomen estar excavado. Las formas con fstula proximal cursan con aspiraciones masivas con la alimentacin. La fstula sin atresia (en H) puede cursar de forma ms larvada y manifestarse como neumonas recurrentes.

Desgloses comentados

509

El tratamiento ha de ser quirrgico. Durante el preoperatorio: posicin en decbito prono, aspiracin continua del bolsn esofgico y medidas generales. La correccin quirrgica suele realizarse en dos tiempos.

Pediatra
P192 (MIR 05-06) Invaginacin intestinal. Imagen anatmica

Tras la ciruga la complicacin ms frecuente ser los trastornos de la motilidad esofgica y, en particular, el reflujo gastroesofgico. Con menos frecuencia aparecen traqueomalacia (respuesta 5), estenosis (respuesta 2) o refistulizacin (respuestas 1 y 3).

P192

MIR 2005-2006

Se trata de una pregunta clsica de Pediatra que no podemos fallar el da del examen. Nos piden el diagnstico de presuncin y nos dan todos los datos tpicos de una invaginacin intestinal, as que aprovecha esta pregunta para repasar bien este cuadro. La edad es tpica, ya que la invaginacin es la causa ms frecuente de obstruccin intestinal entre los tres meses y los seis aos, sobre todo en menores de dos aos, siendo raro en perodo neonatal donde la causa ms habitual de obstruccin intestinal es la enfermedad de Hirschprung. No olvides que ante una invaginacin intestinal en mayores de seis aos se han de descartar causas orgnicas como el linfoma intestinal. La clnica descrita tambin es caracterstica: aparicin brusca de dolor abdominal intenso tipo clico, crisis de llanto, encogimiento de miembros inferiores y palidez cutnea. A medida que avanza el proceso, el nio se encuentra ms dbil, somnoliento y aletargado. En fases iniciales suelen aparecer vmitos. La deposicin de color rojo oscuro es tambin un signo muy tpico de la invaginacin intestinal en las primeras horas de evolucin y se conocen como "heces en jalea de grosella". En fases avanzadas puede cesar la evacuacin de gas y heces y aparecer un estado parecido al shock. Recuerda de cara al MIR que la mejor forma de hacer el diagnstico es la ecografa abdominal, que muestra la imagen en rosquilla, donut o diana en un corte transversal. Otras formas de hacer el diagnstico son la Rx simple de abdomen o el enema opaco con el que observamos el signo del muelle enrollado. Una vez diagnosticada la invaginacin debe procederse a su reduccin hidrosttica siempre que se cumplan dos condiciones: la invaginacin no debe ser prolongada y no debe haber datos de perforacin intestinal. Se podr realizar de dos formas: Reduccin con aire o suero guiada con ecografa: esta opcin asocia menor riesgo de complicaciones (perforacin). Reduccin con enema de bario guiada con control radiolgico: esta opcin presenta diez veces ms de riesgo de perforacin. Si existen signos de perforacin, shock, neumatosis intestinal o distensin abdominal de ms de 48 h de evolucin, es preferible la correccin quirrgica. Por ltimo, recuerda que la forma ms frecuente de invaginacin intestinal es la ileoclica.

P180

MIR 2004-2005

El cuadro que nos comentan sugiere una hernia de Bochdalek. Este tipo de hernia diafragmtica es el ms corriente, especialmente en el lado izquierdo. A travs de la hernia, se produce un paso de rganos abdominales a la cavidad torcica, que es lo que justifica las imgenes areas que nos describen en el hemitrax izquierdo (Rx tpica) (respuesta 4 correcta). Quiz la opcin ms difcil de distinguir sera la malformacin adenoidea qustica, que genera una imagen parecida (pulmn en queso de Gruyre). Pero, en este caso, las imgenes areas seran bilaterales. P180 (MIR 04-05) Hernias diafragmticas

510

Desgloses comentados

P181

Pediatra
MIR 2004-2005
P185 (MIR 04-05) Patogenia de la enfermedad celaca P185 (MIR 04-05) Diagnstico de la enfermedad celaca

Ten mucho cuidado con este tipo de preguntas. Es bastante frecuente que intenten confundirte entre el estreimiento funcional y la enfermedad de Hirschprung. Recuerda que en el estreimiento funcional s que es bastante habitual la presencia de encopresis. Sin embargo, sera muy raro encontrarla en la enfermedad de Hirschprung (respuesta 2 falsa). La rectorragia leve puede estar justificada por el traumatismo de la mucosa rectal en relacin con fenmenos como impactacin y/o heces muy endurecidas. El resto de los datos que nos ofrecen s aparecen en el estreimiento funcional (ver tabla siguiente). P181 (MIR 04-05) Diagnstico diferencial del estreimiento

P174 P185 MIR 2004-2005

MIR 2003-2004

Es una pregunta fcil sabiendo la clnica tpica de la estenosis hipertrfica de ploro EHP. Nos muestran un neonato de 21 das que presenta vmitos postprandiales cada vez ms frecuentes. En un nio de esta edad y con esta clnica nos debemos plantear el diagnstico de EHP. No olvides que el vmito no es bilioso y que el nio queda hambriento e irritable tras el mismo. De entrada podemos descartar la respuesta 4, puesto que la EHP es de etiologa desconocida, pese a que se ha relacionado con factores que predisponen a ello como la administracin de prostaglandina E para mantener la permeabilidad del ductus. Si nos fijamos en la opcin 2, los vmitos persistentes efectivamente generan alteraciones electrolticas, pero no acidosis metablica, sino alcalosis metablica hipoclormica. Si el problema de la EHP es una hiperplasia e hipertrofia de la musculatura lisa del antro gstrico y del duodeno, es lgico pensar que el tratamiento ser quirrgico: pilorotoma de Ramsted (respuesta 3 falsa). Quedan an las opciones 1 y 5. Sabiendo que la EHP es ms habitual en primognitos varones de raza blanca en la tercera semana de vida, podemos afirmar que la 1 es cierta. Recuerda que el polihidramnios no se relaciona con la EHP.

La enfermedad celaca tiene una base inmunopatolgica (respuesta 1 verdadera), habitualmente asintomtica durante un tiempo, con un perodo libre de enfermedad que puede durar aos, hasta que aparecen las primeras manifestaciones clnicas (respuesta 2 cierta). Dentro del diagnstico diferencial, entraran otras enfermedades digestivas, como la giardiasis (respuesta 4 cierta), que afecta tambin a duodeno e leon proximal, compartiendo sntomas como flatulencia, esteatorrea, molestias epigstricas inespecficas, etc. El diagnstico de enfermedad celaca requiere una biopsia de la unin duodeno-yeyunal. La lesin histolgica no es especfica, ya que puede verse en otras enfermedades, pero es muy sugestiva: atrofia de vellosidades e hiperplasia de las criptas (respuesta 5 correcta). No obstante, existen anticuerpos que tambin nos dan una orientacin diagnstica. Los ms especficos son los antitransglutaminasa tisular (respuesta 3 falsa). Repasa con los esquemas siguientes la patogenia y diagnstico de la enfermedad celaca.

Desgloses comentados

511

P174 (MIR 03-04) Estenosis hipertrfica de ploro. Imagen anatmica

Pediatra
P189 MIR 2002-2003 MIR 2003-2004

Supone la causa ms usual de obstruccin intestinal baja en el RN (1/5.000 RN vivos), con mayor incidencia en varones (respuesta 2 verdadera), y puede aparecer asociado a otras alteraciones como sndrome de Down, Lawrence Moon Bield, Waardenburg y defectos cardiovasculares. La clnica ms tpica en el perodo neonatal es el retraso en la eliminacin del meconio. En su diagnstico se emplea la radiografa simple, que muestra distensin de asas con ausencia de aire rectal, el enema opaco, en el que se aprecia un cambio brusco en el dimetro del colon entre el segmento afecto (estenosado) y porcin sana (distendido) (respuesta 5 verdadera); con la manometra anorrectal se observa una contraccin del esfnter anal interno ante el aumento de presin a ese nivel (respuesta 4 falsa), siendo lo normal una relajacin de dicho esfnter. El diagnstico definitivo lo da la biopsia.

Pregunta fcil sobre un tema tpico en el MIR: la estenosis hipertrfica del ploro. Se trata de una pregunta directa sobre las caractersticas de esta enfermedad: Epdemiologa: es ms frecuente en varones (opcin 1 correcta), sobre todo en primognitos. Clnica: el sntoma principal son los vmitos, no biliosos (opcin 3 incorrecta), proyectivos (opcin 4 correcta), que comienzan a partir de los 20 das de vida (opcin 2 correcta). Tras vomitar, el nio queda irritable y hambriento. Tambin son tpicas la alcalosis metablica hipoclormica (debida al vmito) y la ictericia (opcin 5 correcta) (debida a la deshidratacin). Exploracin fsica: palpacin de la oliva pilrica y ondas peristlticas gstricas. Diagnstico: clnica, pH, iones (suficiente en el 60-80%), ecografa abdominal (de eleccin), Rx con bario (signo de la cuerda) y Rx simple (nica burbuja). Diagnstico diferencial: atresia pilrica (vmitos no biliosos desde las primeras tomas con imagen de nica burbuja en la radiografa), atresia duodenal (vmitos biliosos precoces e imagen de doble burbuja en la Rx, regla de la D: duodenal, doble burbuja, sndrome de Down) y reflujo gastroesofgico (vmito atnico de contenido alimentario ya en la primera semana de vida). Tratamiento: preoperatorio: fluidoterapia i.v. para corregir las alteraciones hidroelectrolticas y quirrgico (piloromiotoma de Ramsted).

P175

Nos enfrentamos a una pregunta accesible acerca de la enfermedad de Hirschprung, que podremos contestar fcilmente con conocimientos generales de su clnica tpica. Como sabemos, es una enfermedad congnita (respuesta 1 verdadera) cuyo origen est en una inervacin anmala del intestino, con ausencia de clulas ganglionares del plexo intramural (respuesta 3 verdadera) por una migracin anmala de neuroblastos. P175 (MIR 03-04) Enfermedad de Hirschprung

P196

MIR 2002-2003

Pregunta bastante sencilla sobre la enfermedad celaca, un tema nada infrecuente en el MIR. Si bien la pregunta es en forma de caso clnico, es todo muy tpico y claramente orientador hacia el diagnstico de enfermedad celaca. La opcin falsa es sin duda la 5: el tratamiento de la enfermedad celaca es la retirada total y definitiva (de por vida) del gluten de la dieta, no concibindose su retirada gradual. El resto de opciones nos recuerdan datos relevantes sobre esta enfermedad: necesidad de, al menos, una biopsia (clsicamente 3); la predisposicin gentica (agrupacin familiar, asociacin con ciertos HLA) y su heterogeneidad clnica (tanto en tiempo como formas de presentacin). Repasa la clnica de la enfermedad celaca con el dibujo siguiente.

512

Desgloses comentados

P196 (MIR 02-03) Enfermedad celaca

Pediatra
P201 MIR 2002-2003 MIR 2002-2003

cocitos ni eosinfilos, donde no hay perdida de peso, se tratara de una diarrea crnica inespecfica que afrontaremos tranquilizando a la familia, pues es un cuadro benigno.

Es una pregunta bastante sencilla, de manera que no se puede dudar al contestarla. Forma parte de uno de los temas ms importantes de la pediatra, el aparato digestivo. La aparicin de dolor abdominal brusco intenso de tipo clico, crisis de llanto, encogimiento de miembros inferiores y palidez cutnea, es tpica de la invaginacin intestinal, opcin 4 correcta. La invaginacin es la causa ms frecuente de obstruccin intestinal entre los tres meses y los seis aos, y la localizacin ms comn es a nivel ileoclica. Como aparece en este caso, es tpico la presencia de heces con sangre roja fresca y moco "heces en jalea de grosella". En la exploracin clnica aparece como una masa dolorosa, localizada en hipocondrio derecho. Para confirmar el diagnstico realizaremos un enema o una ecografa, que es la prueba de eleccin, donde veremos la imagen en rosquilla o diana. Respecto al tratamiento, mencionar que se puede intentar reduccin hidrosttica, o si aparecen signos de perforacin intestinal o recidiva se manejar quirrgicamente. En el supuesto de que fuera una estenosis pilrica, la paciente presentara vmitos no biliosos a las dos o tres semanas de vida, sin heces con productos patolgicos.

P200

Es una pregunta que hace referencia a las diarreas crnicas y que tiene una dificultad media. Es un caso clnico donde se nos presenta a una nia que tras un cuadro de gastroenteritis mantiene un proceso diarreico crnico (por durar ms de dos semanas), sin productos patolgicos, con irritacin del rea perianal (que nos indica un pH bajo). Ante este cuadro hemos de sospechar un dficit de disacaridasas secundario a un proceso que ha lesionado de forma difusa el epitelio intestinal, siendo ste el origen ms frecuente. Estos pacientes tambin manifiestan con frecuencia dolor abdominal, borborigmos y, ocasionalmente, dolor clico. Para confirmar el cuadro podemos realizar un clinitest, test de hidrgeno en aire espirado o medicin de pH. El tratamiento estriba en la reduccin de la ingesta de los azcares responsables de la intolerancia a niveles que sean soportables, en este caso de forma transitoria. Por eso les diremos a los padres que alimenten a su hija con frmulas sin lactosa durante varias semanas para que se regenere el epitelio intestinal. Este proceso no requiere tratamiento antibitico, de hecho podra empeorar el cuadro por agresin al epitelio daado, tampoco requiere rehidratacin oral, puesto que en ningn caso se nos menciona que presente prdida de lquidos que justifiquen el tratamiento. Los hidrolizados de protenas seran el tratamiento de eleccin si fuera un paciente que presentara malabsorcin proteica, con prdida de peso, diarrea, hipoproteinemia y edemas. Si la paciente mostrara un aumento del nmero de las deposiciones con moco, fibras vegetales sin digerir y granos de almidn, pero sin sangre, leu-

El resto de las patologas que aparecen se manifiestan de diferente manera, de modo que no plantean dudas de diagnstico diferencial, ya que, por ejemplo, en la apendicitis aparecera un dolor periumbilical sin vmitos ni letargia, adems de no ser frecuente a esta edad.

P180

MIR 2001-2002

El reflujo gastroesofgico (RGE) es un cuadro relativamente corriente en el lactante, debido a diversos factores favorecedores: porcin infradiafragmtica del esfago ms corta, estmago ligeramente ms horizontalizado, menor capacidad gstrica, postura en decbito..., pero no es fisiolgico (respuestas 1 y 4 incorrectas). El sntoma ms frecuente es el vmito, que en la mayora de los casos aparece en la primera semana de vida (recuerda que los vmitos de la estenosis hipertrfica de ploro suelen comenzar hacia la 3 semana). Suele desaparecer hacia los dos aos de vida. El RGE puede presentarse tambin como alguna complicacin: Respiratoria: neumona por aspiracin, tos, sibilancias, etc. Digestiva: esofagitis con hemorragia digestiva, anemia ferropnica, dolor, disfagia, etc. Retraso ponderoestatural. Sndrome de Sandifer: opisttonos y posturas anmalas de la cabeza, en relacin con el RGE, bien como respuesta al dolor o como mecanismo de proteccin de la va area (respuesta 2 correcta). El diagnstico suele realizarse por la clnica y por la respuesta al tratamiento. Slo en las formas ms graves o en ausencia de respuesta al tratamiento emprico se realizan otras pruebas diagnsticas, tales como esofagoscopia

Desgloses comentados

513

y biopsia, esofagograma con bario y pHmetra de 24 horas, siendo esta ltima el mtodo ms sensible (respuesta 3 incorrecta).

Pediatra
P189 MIR 2007-2008 MIR 2001-2002 P193 MIR 2002-2003

El tratamiento del RGE se basa en tres pilares: Medidas generales: espesamiento de las tomas, posicin semiincorporada. Medidas farmacolgicas: procinticos 15-20 minutos antes de tres tomas, anticidos o inhibidores de la bomba de protones si existe esofagitis. Ciruga: cuando no se observa mejora con el tratamiento mdico o aparecen complicaciones severas del RGE se procede a realizar una funduplicatura de Nissen (respuesta 5 incorrecta).

correctamente y no nos indican ninguna otra alteracin en el sedimento, carecen de sentido las opciones 1, 2, 3 y 5, por tanto, la conducta inicial ms adecuada sera repetir el sedimento en 15 das, ya que el aumento de hemates podra ser debido, por ejemplo, a la realizacin de ejercicio fsico intenso, algo muy comn en esta edad.

P186

La invaginacin supone la causa ms frecuente de obstruccin intestinal entre los tres meses y seis aos de edad. Se produce cuando un segmento intestinal se introduce en otro segmento inmediatamente distal a l. La forma ms usual de invaginacin es la ileoclica y la ileoileoclica. La gran mayora de los casos son de etiologa desconocida. Un pequeo porcentaje son secundarios a procesos como las infecciones por adenovirus, divertculo de Meckel, plipos y otros tumores, etc. Clnicamente, se manifiesta con dolor abdominal intenso de tipo clico, encogimiento de miembros inferiores y palidez cutnea. En fases iniciales suelen aparecer vmitos. En las primeras horas de evolucin puede haber eliminacin de heces, pese a que si la invaginacin progresa cesa la evacuacin de gases y heces. Hasta un 60% de los nios afectados pueden presentar heces con sangre roja fresca y moco ("heces en jalea de grosella"). Si progresa, el nio puede entrar en un estado parecido al shock. El diagnstico se establece por la clnica y exploracin fsica (se palpa una masa alargada, dolorosa, localizada en hipocondrio-flanco derecho). En el enema opaco se observa un defecto de replecin al nivel de la cabeza de la invaginacin, y en la radiografa simple de abdomen, un rea de aumento de densidad en hemiabdomen derecho y de distensin de asas en el izquierdo. La prueba fundamental es la ecografa, donde es tpica la imagen en rosquilla o donut. El tratamiento consiste en la reduccin hidrosttica o neumtica bajo control ecogrfico o fluoroscpico. Es preferible la correccin quirrgica en caso de signos de perforacin intestinal, shock, neumatosis intestinal o distensin abdominal de ms de 48 horas de evolucin.

Hasta la edad de los dos o tres aos, los lactantes presentan una fimosis fisiolgica en un altsimo porcentaje de casos (respuesta 1 correcta). Por ello, es evidente que no todos debern ser intervenidos quirrgicamente. Las indicaciones para la circuncisin en menores de un ao son: Que la fimosis produzca infecciones urinarias. Balanopostitis. Parafimosis de repeticin. Fimosis puntiforme (la que deja un orificio prepucial mnimo, de forma que obstaculiza la libre emisin de orina). Es cierto que los corticoides pueden ser tiles en algunos casos y evitar la circuncisin, pero esto no puede entenderse como la regla general, de manera que la respuesta 5 es falsa.

Se trata de una pregunta de dificultad alta sobre un tema que puede aparecer tanto en Pediatra como en Nefrologa o Infecciosas. El sndrome hemoltico urmico, desde el punto de vista peditrico, es la causa ms frecuente de insuficiencia renal aguda en nios menores de cuatro aos. Desde el punto de vista infeccioso destaca que casi siempre va precedido por una gastroenteritis enteroinvasiva por E. coli (si bien tambin se ha relacionado con Shigella, Salmonella y Campylobacter). Desde el punto de vista clnico la trada tpica: anemia hemoltica con esquistocitos, trombopenia con pruebas de coagulacin normales e insuficiencia renal. El principal diagnstico diferencial se establece con la trombosis de la vena renal que cursa con HTA + hematuria + trombopenia. El tratamiento mdico de las manifestaciones hematolgicas, junto con una dilisis peritoneal precoz, es el que mejores expectativas de recuperacin renal ofrece, de tal manera que la funcin renal se recupera en el 90% de los casos y es una enfermedad que no recidiva. De esta manera las pistas que podemos encontrar en este caso clnico son: Cuadro febril con diarrea mucosanguinolenta. Orina hematrica. Anemia. Trombopenia con normalidad de las pruebas de coagulacin. Podemos descartar otras opciones como: Prpura de Schnlein-Henoch: la simple presencia de trombopenia descarta esta opcin, porque es una prpura no trombopnica. Su manifestacin cutnea no son las petequias sino la prpura palpable. Igualmente suele asociar glomerulonefritis, artralgias y dolor abdominal. Glomerulonefritis aguda postinfecciosa: suele aparecer en relacin con infeccin farngea o cutnea por estreptococo del grupo A tras un perodo de latencia, no coincidiendo con la infeccin. Desde el punto de vista renal cursa en forma de sndrome nefrtico (hematuria, HTA y oliguria).

T5
P151

Nefrologa y urologa

MIR 2010-2011

Caso clnico en el que nos presentan una nia de 10 aos con exploracin fsica, talla y TA normales, el anlisis de orina tiene una densidad y un pH dentro de los valores normales, el nico dato alterado es el nmero de hemates por campo (normal 0-4). Puesto que no tiene ningn sntoma (anlisis de orina de rutina), crece

514

Desgloses comentados

P198

Pediatra
MIR 2002-2003 P188 MIR 2001-2002 MIR 2001-2002

Pregunta fcil sobre la infeccin del tracto urinario. Las caractersticas de la ITU en nios son muy similares a las del adulto. En general, es ms genrico en el sexo femenino, la nica excepcin es el primer ao de vida, siendo ms frecuente en este caso en los varones (opcin 1 incorrecta). La causa ms usual de ITU es E. coli (opcin 2 correcta). Tambin podemos encontrar otros gramnegativos como Klebsiella, Proteus... Menos frecuentes son los grampositivos (enterococo, S. epidermidis), adenovirus (cistitis hemorrgica), hongos... La clnica en los nios es bastante inespecfica. Lo que nos puede diferenciar una ITU baja de una alta es la presencia de fiebre en esta ltima. Esta distincin es primordial, sobre todo a la hora del tratamiento: bajaantispticos urinarios, trimetropn-sulfametoxazol o amoxicilina oral; altaaminoglucsidos, cefalosporinas de 3 generacin o amoxiclavulnico inicialmente por va sistmica (opcin 4 correcta). El diagnstico definitivo nos lo da el cultivo. En cuanto a la evaluacin posterior, la ecografa abdominal se indica siempre ante cualquier ITU en un nio (opcin 3 correcta), mientras que la combinacin de eco, cistouretrografa miccional (despistaje de reflujo vesicoureteral y disinergia vesical) y gammagrafa renal con DMSA (mtodo ms sensible para demostrar presencia de cicatrices) (opcin 5 correcta) estara indicada ante toda ITU febril, ITU en menores de cinco aos o en nias en edades posteriores que hayan presentado dos o ms ITU.

Clnicamente, la torsin testicular cursa con dolor agudo y tumefaccin del escroto, exigiendo diagnstico diferencial con la orquiepididimitis. Tambin hay que hacer el diagnstico diferencial con la torsin del hidtide. sta es ms frecuente entre los siete y los 12 aos, suele producir menor dolor y tumefaccin, y ocasionalmente se observa una zona azulada por encima del testculo con sensibilidad aumentada a la palpacin. El tratamiento de la torsin testicular es la ciruga inmediata (respuesta 5 correcta). El procedimiento depende del tiempo de evolucin. Si lleva menos de seis horas, se procede a la detorsin y fijacin al escroto. Despus de seis horas suele ser necesaria la orquiectoma, realizndose tambin la fijacin del testculo contralateral. Repasa el diagnstico diferencial del escroto agudo con la tabla correspondiente a esta pregunta.

El reflujo vesicoureteral se produce como consecuencia de un fallo en el mecanismo valvular que impide el paso de orina desde la vejiga hacia el urter. El dao renal viene determinado tanto por las altas presiones vesicales que se transmiten a los clices renales como por la frecuencia de infecciones, que conducirn finalmente a la existencia de cicatrices renales. El reflujo se clasifica desde el punto de vista etiopatognico en primario o secundario a diversas patologas tales como duplicacin ureteral, divertculo, ureterocele, etc. Existe una clasificacin en cinco grados, segn la altura que alcanza el reflujo y el grado de dilatacin pieloureteral, que tiene implicaciones pronsticas y teraputicas.

P187

El caso clnico nos muestra un cuadro de escroto agudo. La torsin testicular es la causa ms comn de escroto agudo en menores de seis aos, y el primer cuadro a considerar en la edad prepuberal o en el adolescente con dolor escrotal agudo y tumefaccin testicular. P187 (MIR 01-02) Diagnstico diferencial del escroto agudo

Desgloses comentados

515

Una vez diagnosticado, graduado y establecido si el reflujo es primario o secundario, conviene conocer el tamao de los riones y si muestran cicatrices de esclerosis, que se pueden apreciar ya a los cuatro o cinco meses de haber padecido una infeccin urinaria (respuesta 3 correcta); la ecografa permite valorar el tamao renal, y la gammagrafa isotpica del parnquima renal es til para descartar los tractos cicatrizales. La urografa i.v. y la tomografa tambin pueden cumplir estos propsitos. Finalmente, se medir la presin arterial, as como el aclaramiento basal de creatinina.

Pediatra
T6
P187
Hemato-oncologa peditrica

MIR 2005-2006

Pregunta de dificultad media acerca del neuroblastoma. El neuroblastoma es un tumor del sistema nervioso simptico que constituye el 10% de los tumores de la infancia. La patologa oncolgica ms frecuente en la infancia son las leucemias representando el 30%. El tumor slido ms corriente es el tumor cerebral, y dentro de ellos, el astrocitoma. El tumor slido extracraneal ms frecuente es el neuroblastoma. Te ayudar saber el orden de frecuencia (de mayor a menos) de los cnceres en el nio: leucemias, cerebrales, linfomas, neuroblastoma, Wilms, tumores seos, otros (retinoblastoma...). La edad media al diagnstico son los dos aos y el 90% son menores de cinco aos (nuestra pregunta nos habla de un lactante de diez meses). La localizacin ms comn es el abdomen (70%), siendo el 50% de localizacin suprarrenal. P187 (MIR 05-06) Localizacin del neuroblastoma

El tratamiento del reflujo primario difiere segn el grado: En los grados I y II se observa una elevada probabilidad de remisin espontnea. El tratamiento ser expectante, con profilaxis antibitica para mantener estril la orina, y seguimiento trimestral con urocultivo. La ecografa renal anual o bienal tambin resulta til para valorar el crecimiento renal. La gammagrafa renal con DMSA aplicada al concluir el perodo de tratamiento ayuda a seleccionar a los nios con tractos cicatrizales residuales que requieren seguimiento de la presin arterial. El seguimiento de los casos de reflujo grado III es muy parecido, pero se precisan gammagrafas peridicas del parnquima renal si se sospecha la formacin de nuevas cicatrices. Ms de la mitad de los nios con reflujo grado III acaba precisando ciruga. El reflujo grado IV y V requiere ciruga precoz, pues no es la norma la remisin espontnea. El tratamiento del reflujo secundario es el de la causa. Los resultados tras un tratamiento quirrgico son excelentes. P188 (MIR 01-02) Grados del reflujo vesicoureteral

P188 (MIR 01-02) Clasificacin del reflujo

La clnica depende de la localizacin. En la pregunta nos dan el caso tpico de localizacin abdominal que es la ms frecuente y que suele aparecer como una masa abdominal que sobrepasa la lnea media (a diferencia del Wilms). Otras formas de presentacin son:

516

Desgloses comentados

Paravertebral: con clnica de compresin medular. Trax: se suelen descubrir al hacer Rx de trax por otras razones. Cabeza y cuello: pueden dar un sndrome de Horner. Nasofaringe (estesioneuroblastoma): suele manifestarse por epistaxis.

Pediatra
P189 MIR 2001-2002

presencia de metstasis hepticas no contraindica el tratamiento quirrgico. Por ltimo, sealar que la hemihipertrofia es una malformacin congnita que se asocia al tumor de Wilms o nefroblastoma y no al neuroblastoma.

La localizacin ms usual de las metstasis es por va linftica y hemtica a hgado, mdula sea y esqueleto. Consecuentemente, la opcin 3 es correcta, puesto que ante todo diagnstico de neuroblastoma se debe realizar un aspirado de mdula sea como parte del estudio de extensin al ser la mdula una diana frecuente de metstasis. Para el diagnstico se pueden usar: TC abdominal que visualiza el tumor como una masa de densidad mixta slida y qustica, adems de calcificaciones en el 80%. Catecolaminas urinarias elevadas (cido homovalnico, cido vanilmandlico): ste es un dato especfico de este tumor. Gammagrafa con MIBG (metayodobencilguanidina) marcada con istopo radiactivo que capta catecolaminas. Por consiguiente, la opcin 5 es falsa, dado que esta tcnica se sigue usando. Biopsia, que permite el diagnstico anatomopatolgico. Los factores pronsticos, si bien difciles de retener, son esenciales de cara al MIR, puesto que ya han sido objeto de preguntas en otras convocatorias. De buen pronstico: -- Diagnstico en menores de un ao: por tanto, opcin 1 falsa. -- Localizacin en cuello, mediastino posterior o pelvis. -- Vanilmandlico/Homovalnico mayor de 1. -- Aneuploida del ADN (este aspecto ya fue preguntado en exmenes anteriores). De mal pronstico: -- Localizacin abdominal. -- Enolasa srica y/o ferritina elevadas. -- Amplificacin del oncogn n-myc. -- Estadios III y IV. -- Delecin del brazo corto del cromosoma 1. El tratamiento debe ser quirrgico siempre que sea posible , si es necesario administrando previamente quimioterapia en los tumores no resecables. En consecuencia, la opcin 2 es falsa, puesto que la P189 (MIR 01-02) Neuroblastoma frente a nefroblastoma

El tumor de Wilms es el tumor abdominal ms frecuente en la infancia y el tumor renal ms habitual. Puede presentarse aislado o asociado a diversas malformaciones congnitas. Las ms usuales son las anomalas genitourinarias, seguidas de la hemihipertrofia y la aniridia. Se ha asociado a algunos sndromes: sndrome WARG (Wilms, aniridia, retraso mental y malformaciones genitourinarias), sndrome de Dennys Drash (Wilms, retinopata y alteraciones genitourinarias) y el sndrome de Beckwith-Wiedemann. Se pueden encontrar deleciones del cromosoma 11. La edad media de diagnstico de este tumor es hacia los tres aos de edad (la edad media de diagnstico del neuroblastoma es algo menor, hacia los dos aos). La forma ms frecuente de debut es como una masa abdominal asintomtica (tambin es la forma ms comn de presentacin del neuroblastoma). Otras manifestaciones clnicas son la hipertensin arterial (por compresin de la arteria renal por el tumor), hematuria, policitemia por produccin de eritropoyetina y hemorragia intratumoral por traumatismos. La localizacin ms frecuente de metstasis es el pulmn. La TC muestra una masa poco homognea, con zonas de necrosis, hemorragia y calcificaciones focales pequeas (aunque menos corrientes y menos acentuadas que en el neuroblastoma). Si bien la PAAF no ha demostrado aumentar el riesgo de diseminacin del tumor en el trayecto de la puncin, la biopsia no debe hacerse porque la rotura de la cpsula renal cambia el estadio. El tratamiento se establece con ciruga, radioterapia y/o quimioterapia, dependiendo del estadio. Te resultar ms fcil estudiar el neuroblastoma y el nefroblastoma si los comparas. Para ello, puedes utilizar la tabla de la pgina anterior.

Desgloses comentados

517

Pediatra
T7
P231 Enfermedades infecciosas

De acuerdo con los protocolos vigentes, sin la realizacin de pruebas complementarias no quedara descartado el origen orgnico, por lo que la pregunta se anul.

P186 MIR 2011-2012


En esta pregunta se plantea el diagnstico diferencial de enfermedades que cursan con fiebre y exantema. En este caso clnico debes buscar las pistas que te lleven al diagnstico. Lo primero que debe llamar tu atencin son las lesiones blanquecinas en cavidad oral, que no son otras que las famosas manchas de Koplik, que son patognomnicas de sarampin, siendo sta la respuesta correcta. Recuerda que, el sarampin se presenta como un exantema morbiliforme que no blanquea a la presin y puede afectar a palmas y plantas, en el contexto de una infeccin respiratoria con fiebre alta. El hecho de que el nio tenga 7 meses apoya ms el diagnstico, ya que an no ha recibido la vacuna de la triple vrica (cuya primera dosis se administra entre los 12 y los15 meses). Es importante tambin saber por qu no son las otras opciones. En primer lugar, la enfermedad de Kawasaki se diagnostica mediante criterios clnicos fciles de recordar con la regla del ABCDEF. La F de Fiebre de al menos 5 das es el nico criterio imprescindible y debe acompaarse de al menos 4 de los otros 5: A de Adenopata (cervical mayor de 1,5 cm). B de Boca (afectacin de labios, lengua y orofaringe). C de Conjuntivitis (bilateral no exudativa). D de Dedos (eritema, edema y posterior descamacin de manos y pies). E de Exantema. Recuerda que estos sntomas no se pueden justificar por ninguna otra causa. En segundo lugar, no puede tratarse de una escarlatina aunque presente una faringoamigdalitis, ya que en menores de 3 aos stas son prcticamente en su totalidad vricas y recuerda que la causa de la escarlatina es un Streptoccocus pyogenes productor de toxina eritrgena. Fjate adems en que no ha respondido al tratamiento antibitico. En relacin al exantema alrgico por amoxicilina, se presentara como una reaccin de tipo urticarial. Por ltimo, podras pensar que es una mononucleosis por el exantema asociado al empleo de amoxicilina, que es muy caracterstico. No obstante, deberan aportar otros datos como adenopatas mltiples o hepatoesplenomegalia.

MIR 2006-2007

Con respecto de las vas de transmisin vertical del VIH neonatal, recuerda que la tasa de transmisin oscila entre 25-52% y que esta transmisin puede acontecer en tres momentos: va transplacentaria (30-40% de los casos). Va de transmisin durante el parto: va ms importante de transmisin maternofetal (60-70%). Dentro de las variables que facilitan la transmisin destaca la rotura de membranas superior a cuatro horas; otras son la prematuridad y el bajo peso al nacimiento. Va postnatal: existe documentacin de transmisin mediante la leche materna (es una de las pocas contraindicaciones de la lactancia materna, salvo en los pases en vas de desarrollo).

P185

MIR 2005-2006

Pregunta fcil en forma de caso clnico tpico de la enfermedad de Kawasaki. Esta pregunta expone, en forma de caso clnico, la historia de un paciente con enfermedad de Kawasaki. Esta entidad se diagnostica mediante el cumplimiento de sus criterios diagnsticos. Adems de la fiebre, aqu aparecen cuatro: exantema, conjuntivitis bilateral no purulenta, cambios en la mucosa oral (eritema labial, lengua aframbuesada) y adenopatas. La enfermedad acostumbra a presentarse con fiebre alta de ms de cinco das de evolucin que no responde a antibiticos (el nio ha recibido tres dosis de azitromicina). Adems de los cuatro criterios diagnsticos que aparecen en la pregunta, pueden aparecer alteraciones en las zonas perifricas de las extremidades, como edema y eritema en manos o pies, descamacin de inicio periungueal, que suelen aparecer entre la primera y la tercera semana. La manifestacin ms significativa por su gravedad es la afectacin cardaca, que ocurre en un 10-40% de los casos. Se produce vasculitis coronaria en las dos primeras semanas con posterior formacin de aneurismas en cuentas de rosario. Una regla mnemotcnica acerca de las caractersticas de la enfermedad de Kawasaki: ABCDEFG: adenopatas, boca (alteraciones mucosa orofaringe), conjuntivitis, descamacin, exantema, fiebre y gammaglobulina en su tratamiento. Recuerda un dato muy novedoso (ao 2005) acerca de la etiologa de dicha enfermedad: se relaciona con el coronavirus New Haven que actuara como un superantgeno. La clnica del exantema sbito es muy tpica y fcil de reconocer en el MIR. Se produce en nios menores de tres aos. Se caracteriza por fiebre muy alta con aparente buen estado general y sin foco durante tres a cinco das y al tercer o cuarto da la desaparicin brusca de la fiebre coincidiendo con la aparicin de un exantema que no afecta ni a cara ni a extremidades inferiores. Lo causa el VHH tipo 6.

P172

MIR 2009-2010

Este caso sugiere un dolor abdominal recidivante de origen funcional (no orgnico), que es frecuente en la infancia. Tratados de Pediatra, como el de Nelson, sostienen que el estudio de laboratorio no siempre es necesario en una primera visita. Sin embargo, tambin admite como manejo diagnstico inicial la realizacin de un hemograma completo, con VSG, parsitos en heces y anlisis de orina. Por ello, dado que son posibles varias actitudes, se tratara de una pregunta dudosa. Por otra parte, los protocolos de la Asociacin Espaola de Pediatra sostienen que, ante un cuadro de dolor abdominal recurrente debemos cumplir ciertas condiciones para descartar un origen orgnico. Se deben descartar los siguientes signos de alarma: Clnicos (anamnesis y exploracin). Por datos de laboratorio (alteraciones en el hemograma, marcadores inflamatorios elevados, anomalas del sedimento urinario y presencia de sangre oculta en heces).

518

Desgloses comentados

P185 (MIR 05-06) Enfermedades exantemticas y afines

Pediatra

La escarlatina es un cuadro brusco con fiebre y aspecto de enfermedad grave con una lengua en fresa blanca (recubierta por una capa blanquecina) primero y una lengua roja (por descamacin aparece una lengua hipermica). Las amgdalas aparecen cubiertas de un exudado blanco acompaadas de un enantema en el paladar blando. Asimismo, aparece un exantema que predomina en pliegues y que se palpa mejor que se ve (lneas de Pastia). En la cara existe un intenso eritema en frente y mejillas que respeta el tringulo nasogeniano (facies de Filatov). El cuadro est producido por el S. pyogenes. La rubola se caracteriza por una fase prodrmica de catarro leve con fiebre moderada y conjuntivitis sin fotofobia. El signo ms caracterstico es la aparicin 24 h antes del exantema de adenopatas retroauriculares,

cervicales posteriores y postoccipitales. El exantema es morbiliforme, se inicia en zonas retroauriculares y desde all desciende centrfugamente afectando en 24 h a todo el cuerpo, respetando palmas y plantas. La mononucleosis infecciosa afecta tpicamente a sujetos entre 15 y 25 aos y la produce el VEB. El cuadro comienza con sntomas gripales que duran 7-14 das seguidos de un cuadro florido con fiebre alta, dolor farngeo intenso, adenopatas de predomino cervical, hepatoesplenomegalia y rash cutneo maculopapular (sobre todo, en pacientes tratados errneamente con ampicilina al suponer que el cuadro de faringitis es de etiologa bacteriana).

Desgloses comentados

519

P187

Pediatra
MIR 2004-2005 MIR 2003-2004

Las enfermedades exantemticas son un tema relativamente importante. Lo esencial es saber hacer diagnstico diferencial entre ellas. Este caso clnico presenta datos muy tpicos, con lo que el diagnstico no ofrece duda. Fiebre alta + exantema cuando sta desaparece = EXANTEMA SBITO Es muy tpico de esta enfermedad que el exantema respete la cara y extremidades inferiores. Recuerda que el agente causal es el HHV-6. Tambin es caracterstico de esta enfermedad la presencia de leucocitosis con neutrofilia en las primeras 24 horas y su asociacin con las convulsiones febriles.

El SMSL se define como la muerte repentina e inesperada en un lactante por razones que no quedan claras ni despus de la necropsia. Es la forma de muerte ms comn en el primer ao de vida. La mxima incidencia es a los dos o tres meses de edad. Ocurren pocos casos antes de las dos semanas y despus de los seis meses. Los varones tienen ms riesgo que las mujeres. La incidencia es mayor en los meses fros. Se ha supuesto que se produce durante el sueo, pues la mayora de los casos se dan entre la medianoche y las nueve de la maana. Se ha relacionado con mltiples factores biolgicos y epidemiolgicos, entre ellos la posicin en decbito prono y lateral (parece que porque interfieren con el control respiratorio, sobre todo en el momento del despertar).

P172

T9
P259

Maltrato infantil

Se trata de una pregunta fcil que se responde con informacin del Manual CTO. La enfermedad de Kawasaki o sndrome mucocutneo ganglionar es de causa desconocida, pese a que se postula la hiptesis de una toxina que inducira un dao inmunitario endotelial (respuesta 3 verdadera). La enfermedad cursa con fiebre alta de ms de cinco das de evolucin, con mala respuesta a antibiticos (respuesta 2 verdadera) conjuntivitis bilateral, afectacin de mucosa orofarngea y de zonas perifricas de extremidades, fundamentalmente descamacin de inicio periungueal, exantema polimorfo, linfadenopatas cervicales, etc. El diagnstico es, fundamentalmente, clnico (respuesta 1 falsa). La complicacin ms grave es la afectacin coronaria en las dos primeras semanas con aparicin de aneurismas "en cuentas de rosario" (respuesta 4 verdadera) El tratamiento fundamental es gammaglobulina i.v. que previene la afectacin coronaria y salicilatos a dosis altas (respuesta 5 verdadera).

MIR 2008-2009

Una pregunta muy interesante. En ella, intentan confundirnos con infecciones exantemticas como la varicela o la enfermedad de Kawasaki, pero si somos observadores es difcil que nos engaen. Se trata de un nio maltratado, y el dato principal lo tenemos en la piel, que nos permite descartar el resto de las opciones. Una sepsis con CID producira lesiones diseminadas y malestar general. No justificara una distribucin como la descrita, y todava menos un cefalohematoma parietal. La enfermedad de Kawasaki no produce lesiones purpricas, o al menos sera muy infrecuente. Carece, por otra parte, de otros criterios sugestivos (conjuntivitis, adenopatas, etc.). El dficit congnito de antitrombina III produce un estado de trombofilia, es decir, tendencia a la hipercoagulabilidad, mientras que aqu estn describiendo hematomas y equimosis. No existen lesiones vesiculosas (ni vesiculohemorrgicas) que pudieran hacernos pensar en la opcin 4. Tampoco refieren fiebre, prurito, etc. Sin embargo, la caprichosa distribucin de las lesiones y los bordes bien delimitados nos hacen pensar en una causa externa. Si, adems, el nio no est bien nutrido y est muy agitado, habra que considerar un posible maltrato.

T8
P181

Sndrome de la muerte sbita del lactante (SMSL)

MIR 2006-2007

Se consideran factores de riesgo biolgico para el sndrome de la muerte sbita del lactante el sexo masculino, los antecedentes de prematuridad (dos o tres veces ms riesgo), la falta de regulacin trmica, la postura en prono para dormir, mayor frecuencia cardaca en todas las fases del sueo-vigilia, defectos autonmicos, patrones respiratorios con alteraciones en la capacidad de autorreanimacin con peor recuperacin desde una hipoxemia. El QT prolongado tambin favorece la muerte sbita.

T10
P183

Calendario vacunal infantil

MIR 2010-2011

P166

MIR 2003-2004

Pregunta de elevada dificultad sobre un tema bastante preguntado en el examen MIR: las inmunizaciones. La respuesta 1 es correcta, ya que en estudios recientes se ha concluido que la vacunacin antineumoccica masiva de la poblacin infantil menor de 2 aos ha reducido significativamente la tasa de meningitis y enfermedad neumoccica invasiva (por los serotipos incluidos en la va-

Pregunta muy fcil, prcticamente de conocimiento popular. De la muerte sbita, principalmente, hay que saber que se relaciona con el decbito prono y el tabaquismo materno.

520

Desgloses comentados

cuna) en el conjunto de la poblacin. Existen resultados convincentes de que la introduccin de la PCV7 en la poblacin infantil en los EE.UU. ha reducido la incidencia de enfermedad neumoccica invasiva en los nios y en los adultos, por lo que sera esperable que esto ocurriera en otros pases. La respuesta 2 es falsa, ya que la PCV7 tiene su indicacin en nios menores de 2 aos, no en adultos. Las respuestas 3 y 4 son falsas porque hacen referencia a la vacuna polisacrida polivalente (PPV23) que se administra en nios mayores de 2 aos y adultos con enfermedades crnicas, inmunodepresin, mayores de 65 aos y personas institucionalizadas. Tiene una inmunidad poco duradera, por lo que se recomienda en ciertos casos la revacunacin cada 3-5 aos, y no ha demostrado reducir la incidencia de meningitis y enfermedad neumoccica invasiva. La mejora en las condiciones de vida y el mejor uso de los antibiticos (respuesta 5 incorrecta) han influido en la disminucin de la incidencia de meningitis e infecciones invasivas por neumococo pero sin duda, lo ms determinante ha sido la vacunacin con PCV7 en nios menores de 2 aos.

Pediatra
P241 MIR 2005-2006
Pregunta de dificultad media-alta que requiere el conocimiento de patologas maternas que pueden afectar transitoriamente al recin nacido, por lo que debers conocerlas si vuelve a caer. Si nos fijamos bien descubrimos que todas las patologas que nos exponen tienen una etiopatogenia basada en anticuerpos maternos que pueden pasar al feto va transplacentaria, radicando la dificultad en conocer en cul de ellas no aparecen manifestaciones en el recin nacido. En todas pasan estas inmunoglobulinas, pero en una de ellas, la enfermedad celaca en concreto, la aparicin de sntomas no solamente depende de la presencia de stas, sino que requiere otros factores determinantes como el factor gentico y la presencia de gluten en la dieta (no hay gluten en la dieta de un recin nacido) y, asimismo, es necesario conocer que en esta enfermedad existe un intervalo libre de enfermedad que oscila entre tres y seis meses entre la introduccin del gluten y la aparicin de la enfermedad. Por todo eso, sta sera la respuesta correcta. Con relacin al resto de opciones que nos presentan: Si ahora consideramos la respuesta 1 sera falsa, pues en hasta el 15% de hijos de mujeres enfermas o futuras enfermas de miastenia gravis aparecen ya en el primer da postparto sntomas como ptosis palpebral, debilidad generalizada, dificultades de succin... que responden bien al tratamiento y que se recuperan, generalmente, el primer mes. En el caso del hipertiroidismo primario (respuesta 2) tambin sera falsa, pues se ha visto como existe paso transplacentario de inmunoglobulinas estimulantes del tiroides en caso de madres con enfermedad de Graves o tiroiditis de Hashimoto (siendo el propiltiouracilo el tratamiento de eleccin en la embarazada) que causa clnica de tirotoxicosis transitoria. Si la madre tom drogas antitiroideas durante la gestacin, los nios pueden estar asintomticos al nacer y no presentar clnica hasta los cinco o diez das. Finalmente, las respuestas 3 y 4 son enfermedades ampollosas autoinmunes que pueden aparecer en la madre y que en un 5-10% de los casos los anticuerpos pasan la barrera placentaria y provocan durante unas semanas ampollas en el recin nacido (intraepidrmicas en el caso del pnfigo y subepidrmicas en el herpes gestationis), pues estos anticuerpos ocasionan acantolisis al atacar protenas especficas en la piel y en las membranas mucosas.

P185

MIR 2010-2011

Pregunta fcil sobre un tema muy repetido en el examen MIR, las contraindicaciones generales de las vacunas. La respuesta correcta es la 1 debido a que en cuadros con alteraciones inmunitarias importantes (inmunodeficiencias, tratamiento inmunosupresor intenso y prolongado, radioterapia) estn contraindicadas las vacunas de microorganismos vivos atenuados. Por tanto, de las vacunas que aparecen en el enunciado la nica respuesta posible es la triple vrica. El resto son vacunas de toxoides o antgenos purificados, por lo que no suponen riesgo de infeccin para un inmunosuprimido.

Otros temas

P180

MIR 2007-2008

P188

MIR 2004-2005

En ocasiones, los nios pequeos se introducen pequeos objetos en la nariz, en un intento normal de explorar sus propios cuerpos. Lo que se introducen puede ser muy variable: alimentos, semillas, borradores, juguetes. Es un cuadro relativamente frecuente y, slo por la edad del nio, deberas haberte planteado seriamente responder la opcin 4. Por otra parte, la clnica que nos describen encaja bastante bien: Olor ftido o secrecin nasal sanguinolenta. Dificultad para respirar por la fosa nasal afectada. Sensacin de ocupacin en la nariz.

Pregunta muy difcil sobre un tema muy concreto de endocrinologa peditrica. No te preocupes en absoluto si la has fallado. El raquitismo resistente al calcitriol, tambin llamado tipo II, es de herencia autosmica recesiva. Se debe a una resistencia perifrica a las acciones de la vitamina D, en relacin con una anomala de su receptor. Clnicamente, se manifiesta como un sndrome hipotnico y con signos y sntomas de raquitismo severo. Caractersticamente, a diferencia de lo que ocurre en otras carencias de vitamina D, asocia alopecia total.

Desgloses comentados

521

You might also like